Você está na página 1de 68

Histria do Brasil Repblica Repblica Velha, Repblica da Espada, presidncia civil, Poltica dos governadores, caf-com-leite, divises, Aliana

Liberal, coronelismo, Revoluo de 30.

Marechal Deodoro da Fonseca: primeiro presidente do Brasil


Object 1 3 2

Introduo O perodo que vai de 1889 a 1930 conhecido como a Repblica Velha. Este perodo da Histria do Brasil marcado pelo domnio poltico das elites agrrias mineiras, paulistas e cariocas. O Brasil firmou-se como um pas exportador de caf, e a indstria deu um significativo salto. Na rea social, vrias revoltas e problemas sociais aconteceram nos quatro cantos do territrio brasileiro. A Repblica da Espada (1889 a 1894)

Proclamao da Repblica (Praa da Aclimao, atual Praa da Repblica, Rio de Janeiro, 15/11/1889) Em 15 de novembro de 1889, aconteceu a Proclamao da Repblica, liderada pelo Marechal Deodoro da Fonseca. Nos cinco anos iniciais, o Brasil foi governado por militares. Deodoro da Fonseca, tornou-se Chefe do Governo Provisrio. Em 1891, renunciou e quem assumiu foi o vicepresidente Floriano Peixoto. O militar Floriano, em seu governo, intensificou a represso aos que ainda davam apoio monarquia. A Constituio de 1891 ( Primeira Constituio Republicana) Aps o incio da Repblica havia a necessidade da elaborao de uma nova Constituio, pois a antiga ainda seguia os ideais da monarquia. A constituio de 1891, garantiu alguns avanos polticos, embora apresentasse algumas limitaes, pois representava os interesses das elites agrrias do pais. A nova constituio implantou o voto universal para os cidados ( mulheres, analfabetos, militares de baixa patente ficavam de fora ). A constituio instituiu o presidencialismo e o voto aberto. Repblica das Oligarquias O perodo que vai de 1894 a 1930 foi marcado pelo governo de presidentes civis, ligados ao setor agrrio. Estes polticos saiam dos seguintes partidos: Partido Republicano Paulista (PRP) e Partido Republicano Mineiro (PRM). Estes dois partidos controlavam as eleies, mantendo-se no poder de maneira alternada. Contavam com o apoio da elite agrria do pas. Dominando o poder, estes presidentes implementaram polticas que beneficiaram o setor agrrio do pas, principalmente, os fazendeiros de caf do oeste paulista. Surgiu neste perodo o tenentismo, que foi um movimento de carter poltico-militar, liderado por tenentes, que faziam oposio ao governo oligrquico. Defendiam a moralidade poltica e mudanas no sistema eleitoral (implantao do voto secreto) e transformaes no ensino pblico do pas. A Coluna Prestes e a Revolta dos 18 do Forte de Copacabana foram dois exemplos do movimento tenentista. Poltica do Caf-com-Leite A maioria dos presidentes desta poca eram polticos de Minas Gerais e So Paulo. Estes dois estados eram os mais ricos da nao e, por isso, dominavam o cenrio poltico da repblica. Sados das elites mineiras e paulistas, os presidentes acabavam favorecendo sempre o setor agrcola, principalmente do caf (paulista) e do leite (mineiro). A poltica do caf-com-leite sofreu duras crticas

de empresrios ligados indstria, que estava em expanso neste perodo. Se por um lado a poltica do caf-com-leite privilegiou e favoreceu o crescimento da agricultura e da pecuria na regio Sudeste, por outro, acabou provocando um abandono das outras regies do pas. As regies Nordeste, Norte e Centro-Oeste ganharam pouca ateno destes polticos e tiveram seus problemas sociais agravados. Poltica dos Governadores Montada no governo do presidente paulista Campos Salles, esta poltica visava manter no poder as oligarquias. Em suma, era uma troca de favores polticos entre governadores e presidente. O presidente apoiava os candidatos dos partidos governistas nos estados, enquanto estes polticos davam suporte a candidatura presidencial e tambm durante a poca do governo. O coronelismo A figura do "coronel" era muito comum durante os anos iniciais da Repblica, principalmente nas regies do interior do Brasil. O coronel era um grande fazendeiro que utilizava seu poder econmico para garantir a eleio dos candidatos que apoiava. Era usado o voto de cabresto, em que o coronel (fazendeiro) obrigava e usava at mesmo a violncia para que os eleitores de seu "curral eleitoral" votassem nos candidatos apoiados por ele. Como o voto era aberto, os eleitores eram pressionados e fiscalizados por capangas do coronel, para que votasse nos candidatos indicados. O coronel tambm utilizava outros "recursos" para conseguir seus objetivos polticos, tais como: compra de votos, votos fantasmas, troca de favores, fraudes eleitorais e violncia. O Convnio de Taubat Essa foi uma frmula encontrada pelo governo republicano para beneficiar os cafeicultores em momentos de crise. Quando o preo do caf abaixava muito, o governo federal comprava o excedente de caf e estocava. Esperava-se a alta do preo do caf e ento os estoques eram liberados. Esta poltica mantinha o preo do caf, principal produto de exportao, sempre em alta e garantia os lucros dos fazendeiros de caf. A crise da Repblica Velha e o Golpe de 1930 Em 1930 ocorreriam eleies para presidncia e, de acordo com a poltica do caf-com-leite, era a vez de assumir um poltico mineiro do PRM. Porm, o Partido Republicano Paulista do presidente Washington Lus indicou um poltico paulista, Julio Prestes, a sucesso, rompendo com o caf-comleite. Descontente, o PRM junta-se com polticos da Paraba e do Rio Grande do Sul (forma-se a Aliana Liberal ) para lanar a presidncia o gacho Getlio Vargas. Jlio Prestes sai vencedor nas eleies de abril de 1930, deixando descontes os polticos da Aliana Liberal, que alegam fraudes eleitorais. Liderados por Getlio Vargas, polticos da Aliana Liberal e militares descontentes, provocam a Revoluo de 1930. o fim da Repblica Velha e incio da Era

Vargas.
Galeria dos Presidentes da Repblica Velha: Marechal Deodoro da Fonseca (15/11/1889 a 23/11/1891), Marechal Floriano Peixoto (23/11/1891 a 15/11/1894), Prudente Moraes (15/11/1894 a 15/11/1898), Campos Salles (15/11/1898 a 15/11/1902) , Rodrigues Alves (15/11/1902 a 15/11/1906), Affonso Penna (15/11/1906 a 14/06/1909), Nilo Peanha (14/06/1909 a 15/11/1910), Marechal Hermes da Fonseca (15/11/1910 a 15/11/1914), Wenceslau Brz (15/11/1914 a 15/11/1918), Delfim Moreira da Costa Ribeiro (15/11/1918 a 27/07/1919), Epitcio Pessoa (28/07/1919 a 15/11/1922), Artur Bernardes (15/11/1922 a 15/11/1926), Washington Luiz (15/11/1926 a 24/10/1930). Voc sabia? - O perodo da Histria do Brasil conhecido como Nova Repblica teve incio em 1985, com o fim da Ditadura Militar e incio do processo de redemocratizao. Este perodo da Histria do Brasil dura at os dias atuais. - A palavra Repblica tem origem no latim res publica, cujo significado "coisa pblica". Guerra do Contestado A Histria da Guerra do Contestado, causas da revolta, monge Jos Maria, incio dos conflitos e o fim da Guerra

Rebeldes armados que participaram da Guerra do Contestado (fonte: Museu Paranaense)

Introduo A Guerra do Contestado foi um conflito armado que ocorreu na regio Sul do Brasil, entre outubro de 1912 e agosto de 1916. O conflito envolveu cerca de 20 mil camponeses que enfrentaram foras militares dos poderes federal e estadual. Ganhou o nome de Guerra do Contestado, pois os conflitos ocorrem numa rea de disputa territorial entre os estados do Parar e Santa Catarina. Causas da Guerra A estrada de ferro entre So Paulo e Rio Grande do Sul estava sendo construda por uma empresa norte-americana, com apoio dos coronis (grandes proprietrios rurais com fora poltica) da regio e do governo. Para a construo da estrada de ferro, milhares de famlia de camponeses perderam suas terras. Este fato, gerou muito desemprego entre os camponeses da regio, que ficaram sem terras para trabalhar. Outro motivo da revolta foi a compra de uma grande rea da regio por de um grupo de pessoas ligadas empresa construtora da estrada de ferro. Esta propriedade foi adquirida para o estabelecimento de uma grande empresa madeireira, voltada para a exportao. Com isso, muitas famlias foram expulsas de suas terras. O clima ficou mais tenso quando a estrada de ferro ficou pronta. Muitos trabalhadores que atuaram em sua construo tinham sido trazidos de diversas partes do Brasil e ficaram desempregados com o fim da obra. Eles permaneceram na regio sem qualquer apoio por parte da empresa norteamericana ou do governo. Participao do monge Jos Maria Nesta poca, as regies mais pobres do Brasil eram terreno frtil para o aparecimento de lideranas religiosas de carter messinico. Na rea do Contestado no foi diferente, pois, diante da crise e insatisfao popular, ganhou fora a figura do beato Jos Maria. Este pregava a criao de um mundo novo, regido pelas leis de Deus, onde todos viveriam em paz, com prosperidade justia e terras para trabalhar. Jos Maria conseguiu reunir milhares de seguidores, principalmente de camponeses sem terras. Os conflitos Os coronis da regio e os governos (federal e estadual) comearam a ficar preocupados com a liderana de Jos Maria e sua capacidade de atrair os camponeses. O governo passou a acusar o beato de ser um inimigo da Repblica, que tinha como objetivo desestruturar o governo e a ordem da regio. Com isso, policiais e soldados do exrcito foram enviados para o local, com o objetivo de desarticular o movimento. Os soldados e policiais comearam a perseguir o beato e seus seguidores. Armados de espingardas de caa, faces e enxadas, os camponeses resistiram e enfrentaram as foras oficiais que estavam bem armadas. Nestes conflitos armados, entre 5 mil e 8 mil rebeldes, na maioria camponeses, morreram. As baixas do lado das tropas oficiais foram bem menores. O fim da Guerra A guerra terminou somente em 1916, quando as tropas oficiais conseguiram prender Adeodato, que era um dos chefes do ltimo reduto de rebeldes da revolta. Ele foi condenado a trinta anos de priso. Concluso A Guerra do Contestado mostra a forma com que os polticos e os governos tratavam as questes sociais no incio da Repblica. Os interesses financeiros de grandes empresas e proprietrios rurais ficavam sempre acima das necessidades da populao mais pobre. No havia espao para a tentativa de solucionar os conflitos com negociao. Quando havia organizao daqueles que eram injustiados, as foras oficiais, com apoio dos coronis, combatiam os movimentos com represso e fora militar. Situao do Nordeste no final do sculo XIX (contexto histrico) - Fome desemprego e baixssimo rendimento das famlias deixavam muitos sem ter o que comer; - Seca a regio do agreste ficava muitos meses e at anos sem receber chuvas. Este fator dificultava a agricultura e matava o gado. - Falta de apoio poltico os governantes e polticos da regio no davam a mnima ateno para as populaes carentes;

- Violncia era comum a existncia de grupos armados que trabalhavam para latifundirios. Estes espalhavam a violncia pela regio. - Desemprego grande parte da populao pobre estava sem emprego em funo da seca e da falta de oportunidades em outras reas da economia. - Fanatismo religioso: era comum a existncia de beatos que arrebanhavam seguidores prometendo uma vida melhor. Dados da Guerra de Canudos: - Perodo: de novembro de 1896 a outubro de 1897. - Local: interior do serto da Bahia - Envolvidos: de um lado os habitantes do Arraial de Canudos (jagunos, sertanejos pobres e miserveis, fanticos religiosos) liderados pelo beato Antnio Conselheiro. Do outro lado as tropas do governo da Bahia com apoio de militares enviados pelo governo federal. Causas da Guerra: O governo da Bahia, com apoio dos latifundirios, no concordavam com o fato dos habitantes de Canudos no pagarem impostos e viverem sem seguir as leis estabelecidas. Afirmavam tambm que Antnio Conselheiro defendia a volta da Monarquia. Por outro lado, Antnio Conselheiro defendia o fim da cobrana dos impostos e era contrrio ao casamento civil. Ele afirma ser um enviado de Deus que deveria liderar o movimento contra as diferenas e injustias sociais. Era tambm um crtico do sistema republicano, como ele funcionava no perodo. Os conflitos militares Nas trs primeiras tentativas das tropas governistas em combater o arraial de Canudos nenhuma foi bem sucedida. Os sertanejos e jagunos se armaram e resistiram com fora contra os militares. Na quarta tentativa, o governo da Bahia solicitou apoio das tropas federais. Militares de vrias regies do Brasil, usando armas pesadas, foram enviados para o serto baiano. Massacraram os habitantes do arraial de Canudos de forma brutal e at injusta. Crianas, mulheres e idosos foram mortos sem piedade. Antnio Conselheiro foi assassinado em 22 de setembro de 1897. Significado do conflito A Guerra de canudos significou a luta e resistncia das populaes marginalizadas do serto nordestino no final do sculo XIX. Embora derrotados, mostraram que no aceitavam a situao de injustia social que reinava na regio. Revolta da Chibata A Histria da Revolta da Chibata, causas, reivindicaes dos marinheiros, acontecimentos, lder Joo Cndido (Almirante Negro), punio para os revoltosos

Joo Cndido (Almirante Negro): lder da revolta

Introduo A Revolta da Chibata foi um importante movimento social ocorrido, no incio do sculo XX, na cidade do Rio de Janeiro. Comeou no dia 22 de novembro de 1910. Neste perodo, os marinheiros brasileiros eram punidos com castigos fsicos. As faltas graves eram punidas com 25 chibatadas (chicotadas). Esta situao gerou uma intensa revolta entre os marinheiros. Causas da revolta O estopim da revolta ocorreu quando o marinheiro Marcelino Rodrigues foi castigado com 250 chibatadas, por ter ferido um colega da Marinha, dentro do encouraado Minas Gerais. O navio de guerra estava indo para o Rio de Janeiro e a punio, que ocorreu na presena dos outros marinheiros, desencadeou a revolta. O motim se agravou e os revoltosos chegaram a matar o comandante do navio e mais trs oficiais. J na Baia da Guanabara, os revoltosos conseguiram o apoio dos marinheiros do encouraado So

Paulo. O clima ficou tenso e perigoso. Reivindicaes O lder da revolta, Joo Cndido (conhecido como o Almirante Negro), redigiu a carta reivindicando o fim dos castigos fsicos, melhorias na alimentao e anistia para todos que participaram da revolta. Caso no fossem cumpridas as reivindicaes, os revoltosos ameaavam bombardear a cidade do Rio de Janeiro (ento capital do Brasil). Segunda revolta Diante da grave situao, o presidente Hermes da Fonseca resolveu aceitar o ultimato dos revoltosos. Porm, aps os marinheiros terem entregues as armas e embarcaes, o presidente solicitou a expulso de alguns revoltosos. A insatisfao retornou e, no comeo de dezembro, os marinheiros fizeram outra revolta na Ilha das Cobras. Esta segunda revolta foi fortemente reprimida pelo governo, sendo que vrios marinheiros foram presos em celas subterrneas da Fortaleza da Ilha das Cobras. Neste local, onde as condies de vida eram desumanas, alguns prisioneiros faleceram. Outros revoltosos presos foram enviados para a Amaznia, onde deveriam prestar trabalhos forados na produo de borracha. O lder da revolta Joo Cndido foi expulso da Marinha e internado como louco no Hospital de Alienados. No ano de 1912, foi absolvido das acusaes junto com outros marinheiros que participaram da revolta. Concluso: podemos considerar a Revolta da Chibata como mais uma manifestao de insatisfao ocorrida no incio da Repblica. Embora pretendessem implantar um sistema poltico-econmico moderno no pas, os republicanos trataram os problemas sociais como casos de polcia. No havia negociao ou busca de solues com entendimento. O governo quase sempre usou a fora das armas para colocar fim s revoltas, greves e outras manifestaes populares. O Cangao A Histria do Cangao, os cangaceiros, Lampio, problemas sociais no Nordeste no incio do sculo XX, fim do cangao

Lampio: o "rei do Cangao"

Introduo

Entre o final do sculo XIX e comeo do XX (incio da Repblica), surgiu, no nordeste brasileiro, grupos de homens armados conhecidos como cangaceiros. Estes grupos apareceram em funo, principalmente, das pssimas condies sociais da regio nordestina. O latifndio, que concentrava terra e renda nas mos dos fazendeiros, deixava as margens da sociedade a maioria da populao. Entendendo o cangao Portanto, podemos entender o cangao como um fenmeno social, caracterizado por atitudes violentas por parte dos cangaceiros. Estes, que andavam em bandos armados, espalhavam o medo pelo serto nordestino. Promoviam saques a fazendas, atacavam comboios e chegavam a seqestrar fazendeiros para obteno de resgates. Aqueles que respeitavam e acatavam as ordens dos cangaceiros no sofriam, pelo contrrio, eram muitas vezes ajudados. Esta atitude, fez com que os cangaceiros fossem respeitados e at mesmo admirados por parte da populao da poca. Os cangaceiros no moravam em locais fixos. Possuam uma vida nmade, ou seja, viviam em movimento, indo de uma cidade para outra. Ao chegarem nas cidades pediam recursos e ajuda aos moradores locais. Aos que se recusavam a ajudar o bando, sobrava a violncia. Como no seguiam as leis estabelecidas pelo governo, eram perseguidos constantemente pelos policiais. Usavam roupas e chapus de couro para protegerem os corpos, durante as

fugas, da vegetao cheia de espinhos da caatinga. Alm desse recurso da vestimenta, usavam todos os conhecimentos que possuam sobre o territrio nordestino (fontes de gua, ervas, tipos de solo e vegetao) para fugirem ou obterem esconderijos. Existiram diversos bandos de cangaceiros. Porm, o mais conhecido e temido da poca foi o comandado por Lampio (Virgulino Ferreira da Silva), tambm conhecido pelo apelido de Rei do Cangao. O bando de Lampio atuou pelo serto nordestino durante as dcadas de 1920 e 1930. Morreu numa emboscada armada por uma volante, junto com a mulher Maria Bonita e outros cangaceiros, em 29 de julho de 1938. Tiveram suas cabeas decepadas e expostas em locais pblicos, pois o governo queria assustar e desestimular esta prtica na regio. Depois do fim do bando de Lampio, os outros grupos de cangaceiros, j enfraquecidos, foram se desarticulando at terminarem de vez ,no final da dcada de 1930.

Historia testes
xerciciosO Brasil mudou muito com a proclamao da repblica. Conhea os principais acontecimentos com estes exerccios (UFMG) Leia o texto: MESTRE-SALA DOS MARES Joo Bosco e Aldir Blanc H muito tempo na guas da Guanabara o drago do mar reapareceu na figura de um bravo feiticeiro a quem a histria no esqueceu conhecido como navegante negro tinha a dignidade de um mestre-sala e ao acenar pelo mar na alegria das regatas foi saudado no porto pelas mocinhas francesas jovens polacas e por batalhes de mulatas rubras cascatas jorravam das costas dos centros entre cantos e chibatas inundando corao, do pessoal do poro e a exemplo do feiticeiro gritava ento Glria aos piratas, s mulatas, s sereias Glria farofa, cachaa, s baleias Glria a todas as lutas inglrias que atravs da nossa histria no esquecemos jamais Salve o navegante negro que tem por monumento as pedras pisadas do cais. A msica se refere ao movimento dos marinheiros brasileiros conhecido como Revolta da Chibata, em reao aos castigos fsicos e s condies de trabalho degradantes. Assinale a alternativa diretamente referida ao contexto em que ocorreu esse movimento. A avaliao, pelos meios operrios, dos soldados e marinheiros como aliados em uma revoluo social. A inviabilidade do exerccio da represso por parte do governo central aos movimentos populares no incio da Repblica. A regulamentao pelo poder militar das relaes conflituosas entre os ex-escravos e seus patres.

A adoo de uma poltica institucional para veicular uma imagem de populao ordeira e de uma sociedade sem conflitos nos primeiros anos da Repblica. A facilidade de ascenso social e hierrquica aos marinheiros solidificou a permanncia do regime imperial. Histria - Repblica Velha - 10 questes O Brasil mudou muito com a proclamao da repblica. Conhea os principais acontecimentos com estes exerccios (Uece) "A dcada de 1920 terminou presenciando uma das poucas campanhas eleitorais da Primeira Repblica em que houve autntica competio para o cargo da Presidncia". (FONTE: CARVALHO, Jos Murilo. Marco Divisrio. In "Cidadania no Brasil: o longo caminho". Rio de Janeiro: Civilizao Brasileira, 2001, pp.89-126.) Assinale a alternativa que contm os nomes dos dois candidatos que disputaram a Presidncia da Repblica, na ocasio. Washington Luis e Getlio Vargas. Washington Luis e Jlio Prestes. Hermes da Fonseca e Getulio Vargas. Getlio Vargas e Jlio Prestes. Rodrigues Alves e Lus Carlos Prestes. O Brasil mudou muito com a proclamao da repblica. Conhea os principais acontecimentos com estes exerccios (PUC-RS - 2007)A poltica econmica de Rui Barbosa foi inadequada ao contexto socioeconmico brasileiro. No incio da Repblica Velha, este "engano" estratgico da poltica do Encilhamento provocou inflao e muitas falncias, tudo em nome do projeto de desenvolvimento industrial. O desastre desta poltica econmica ocorreu porque: o Brasil possua muitas reservas de capital, um mercado interno insuficiente para acompanhar o processo de industrializao e recebia estmulo de outras naes industrializadas. no possuindo reservas de capital e apresentando um mercado interno insuficiente para acompanhar o processo de industrializao, o Brasil recebia presso dos pases industrializados para impedir a concorrncia. embora o Brasil no possusse reservas de capital, tinha um mercado interno capaz de acompanhar o processo de industrializao, porm recebia presso dos pases industrializados, para evitar a concorrncia. o Brasil no possua reservas de capital, possua um mercado interno apto para o consumo e produo agrcola e manufatureira, mas no recebia apoio dos pases industrializados. o Brasil possua muitas reservas de capital, provenientes da economia cafeeira, o que desenvolveu o mercado interno e conquistou o apoio dos pases industrializados e em vias de industrializao. (UFPR)Ora entendidos como bandidos, ora como verdadeiros heris, no incio do sculo XX, homens e mulheres das classes populares impunham suas leis e afrontavam o poder no Nordeste brasileiro, sendo destacados na histria, na literatura e no cinema. Ainda hoje so forte referncia no cancioneiro popular. Sobre esse movimento popular e seus integrantes, correto afirmar: Defendiam o movimento integralista, cujo objetivo era o povoamento efetivo dos sertes.

Eram chamados de cangaceiros, e seu movimento caracterizava-se como uma forma de banditismo social. Seus membros realizavam protestos contra a mecanizao da agricultura e a monocultura. Seguiam um lder messinico que defendia o retorno da monarquia e o comunismo agrrio. Tratava-se de um movimento separatista que recusava a hegemonia da regio Sul. "Cabo de enxada engrossa as mos - o lao de couro cru, machado e foice tambm. Caneta e lpis so ferramentas muito delicadas. A lida outra: labuta pesada, de sol a sol, nos campos e nos currais (...) Ler o qu? Escrever o qu? Mas agora preciso: a eleio vem a e o alistamento rende a estima do patro, a gente vira pessoa." (Palmrio, Mario. VILA DOS CONFINS) A respeito da Repblica Velha: foi um perodo de grande instabilidade poltica devido s diversas manifestaes de trabalhadores rurais que ocorreram e tinham o propsito de acabar com o sistema eleitoral fraudulento. as campanhas eleitorais tinham o objetivo de manipular os eleitores urbanos, pois estes lutavam por conseguir um pedao de terra e se tornarem latifundirios. havia um grande predomnio das oligarquias rurais sobre as populaes, sendo os trabalhadores subordinados a votar nos candidatos dos coronis. as oligarquias buscavam apoio para seus candidatos nas classes que eram alfabetizadas e que no tinham sido maltratadas pelo trabalho no campo. a poltica estava vinculada ao sufrgio universal e secreto, criando desta forma um campo amplo de democracia para todos, principalmente para o trabalhador rural. O Brasil mudou muito com a proclamao da repblica. Conhea os principais acontecimentos com estes exerccios "A Coluna Prestes tendo adotado a ttica da 'guerra de movimento' no s garantiu a prpria sobrevivncia em condies que lhe eram extremamente desfavorveis, como se transformou num exrcito com caractersticas populares cuja marcha pelo Brasil foi decisiva para que se mantivesse acesa a chama da revoluo tenentista". (Anita Leocdia Prestes). Dentro das manifestaes tenentistas, a Coluna Prestes tinha como finalidade: apoiar economicamente as oligarquias cafeeiras. acabar com a repblica oligrquica e instituir uma ditadura militar. instalar o socialismo no Brasil. moralizar a poltica brasileira atravs do voto secreto. mobilizar e conscientizar a classe operria de seus direitos. Questes: Nas questes de 01 a 05 utilize o cdigo abaixo:

a) I, II e III so corretas b) I, II e III so incorretas c) I e II so corretas d) I e III so corretas e) II e III so corretas 01. (UFGO) I. A suspenso dos alvars que proibiam as manufaturas no Brasil permitiu que o pas tivesse um considervel desenvolvimento industrial. II. A pequena dimenso do mercado interno brasileiro e o baixo poder aquisitivo da populao foi fatores que tolheram o desenvolvimento industrial brasileiro. III. O grande momento no processo industrial brasileiro foi a II Guerra Mundial, quando se instaurou um sistema que significava mudana na estrutura da economia, principalmente em seu aspecto qualitativo. 02. (MACK) I. As faces liberal e realista da poca da independncia brasileira conciliaram suas divergncias para organizar e manter a unidade poltica do pas. II. Segundo alguns historiadores, Deodoro e Floriano desempenharam papel de simples substitutos do Poder Moderador, na mesma tradio centralizadora do Imprio, sem alterar as estruturas do pas. III. Os ressentimentos dos oficiais com a chamada Questo Militar, de 1884 1885, foram capitalizados em prol da causa republicana. 03. (UnB) I. A principal caracterstica da economia brasileira, segundo Celso Furtado, na primeira metade do sculo XX, a emergncia de um sistema cujo principal centro dinmico o mercado interno. II. Ao desenvolvimento industrial brasileiro que sucede prosperidade cafeeira, corresponde uma acentuada concentrao regional de renda. III. A integrao do Nordeste economia industrializada obedece a um planejamento prioritrio que se iniciou no governo Vargas. 04. (PUCC) I. A crescente procura de reas favorveis ao cultivo do caf contribuiu para o povoamento da costa paulistana, em princpios do sculo XX. II. O excesso de produo cafeeira agravou os problemas financeiros da Repblica Velha. III. A decadncia do caf nas regies do Vale do Paraba se iniciou a partir da queda da Bolsa em 1929. 05. (FUVEST) I. O debate sucessrio de 1910 se caracterizou pela reao s candidaturas oficiais. II. As dissenses entre os grupos militares e oligarquia tradicional, que apoiaram a candidatura Hermes da Fonseca, culminaram na intranqilidade poltica que caracterizou seu quadrinio. III. O grupo mineiro do Jardim da Infncia representou, no governo Afonso Pena, a reao ao Bloco de Pinheiro Machado.

06. (PUC) A Rebelio de Canudos foi fruto: a) Do fanatismo religioso de populares sem condies econmicas de subsistncia; b) Do desejo de restaurar a monarquia portuguesa no Brasil; c) Da conspirao de grupos conservadores; d) Da organizao de grupos de jagunos no serto; e) n.d.a. 07. (FMU) Rui Barbosa teve atuao destacada como ministro da Fazenda do Governo Provisrio. Entre as medidas que implantou salienta-se: a) Ampliao do crdito lavoura, com indenizao aos donos de escravos, em conseqncia da abolio; b) Reforma do sistema de crdito, com incentivo ao setor industrial; c) Poltica tarifria, estimulando a importao de bens de consumo interno; d) Organizao da legislao de sociedades annimas, visando atrair investimentos estrangeiros no setor industrial. 08. (PUC) A base da economia brasileira durante a Primeira Repblica foi o caf e isto se deveu: a) mudana de regime poltico, liberdade de ao dada aos proprietrios pela Constituio e aos assalariados italianos; b) Ao incentivo dado aos plantadores de caf, aceitao do nosso produto pela Inglaterra e libertao dos escravos; c) decadncia da industrializao, Guerra de Secesso dos Estados Unidos e decadncia da minerao; d) qualidade das terras, ao clima favorvel, imigrao europia e aceitao do nosso produto no mercado externo. e) n.d.a. 09. (PUC) O tenentismo constituiu um dos elementos bsicos: a) Da revoluo brasileira de 1930; b) Da guerra contra Rosas e Oribe; c) Da guerra do Paraguai; d) Da Questo Militar do II Reinado; e) n.d.a. 10. (FACULDADES OBJETIVO) A partir da Revoluo de 1930, desenvolveu-se definitivamente um novo setor na economia brasileira: a) caf b) indstria urbana c) indstria do acar d) exportao e) n.d.a.

Resoluo: 01. E 02. A 03. C 04. C 05. A 06. A 07. B 08. D

09. A 10. B Questes: 01. (MACKENZIE) Do ponto de vista poltico, podemos considerar o Perodo Regencial como: a) uma poca conturbada politicamente, embora sem lutas separatistas que comprometessem a unidade do pas; b) um perodo em que as reivindicaes populares, como direito de voto, abolio da escravido e descentralizao poltica, foram amplamente atendidas; c) uma transio para o regime republicano que se instalou no pas a partir de 1840; d) uma fase extremamente agitada com crises e revoltas em vrias provncias, geradas pelas contradies daselites, classe mdia e camadas populares; e) uma etapa marcada pela estabilidade poltica, j que a oposio ao Imperador Pedro I aproximou os vrios segmentos sociais, facilitando as alianas na Regncia. 02. Durante o Perodo Regencial: a) A monarquia imperial foi extinta, instaurando-se em seu lugar uma repblica Federalista. b) Os regentes governaram de forma absoluta, fazendo uso indiscriminado do Poder Moderador. c) As faces federalistas criaram a Guarda Nacional, um eficiente instrumento militar de oposio ao Exrcito regular da Regncia. d) Nenhum regente fez uso do Poder Moderador, o que, de certa maneira, permitiu a prtica do Parlamentarismo. e) As camadas populares defenderam a proclamao de Repblica e a extino da escravido. 03. (UFGO) O Perodo Regencial apresentou as seguintes caractersticas, menos: a) Durante as Regncias surgiram nossos primeiros partidos polticos: o Liberal e o Conservador. b) O Partido Liberal representava as novas aspiraes populares, revolucionrias e republicanas. c) Foi um perodo de crise econmica e social que resultou em revolues como a Cabanagem e a Balaiada. d) Houve a promulgao do Ato Adicional Constituio, pelo qual o regente passaria a ser eleito diretamente pelos cidados com direito de voto. e) Formaram-se as lideranas polticas que teriam atuao marcante no II Reinado. 04. (UNITAU) Sobre o Perodo Regencial (1831 - 1840), incorreto afirmar que: a) foi um perodo de intensa agitao social, com a Cabanagem no Rio Grande do Sul e a guerra dos Farrapos no Rio de Janeiro; b) passou por trs etapas: regncia trina provisria, regncia trina e regncia una; c) foi criada a Guarda Nacional, formada por tropas controladas pelos grandes fazendeiros; d) atravs do Ato Adicional as provncias ganharam mais autonomia; e) cai a participao do acar entre os produtos exportados pelo Brasil e cresce a participao do caf. 05. (UFS) " ... desligado o povo rio-grandense da comunho brasileira, reassume todos os direitos da primitiva liberdade; usa destes direitos imprescritveis constituindo-se Repblica Independente; toma na

extensa escala dos Estados Soberanos o lugar que lhe compete ..." Na evoluo histrica brasileira, pode-se associar as idias do texto : a) Sabinada b) Balaiada c) Farroupilha d) Guerra dos Emboabas e) Confederao do Equador 06. "Em 1835, o temor da "haitianizao" que j era comum entre muitos polticos do Primeiro Reinado, cresceu ainda mais depois da veiculao da estarrecedora notcia: milhares de escravos se amotinaram a ameaavam tomar a capital da provncia." O texto acima trata da: a) Balaiada ocorrida no Maranho; b) Revolta dos Quebra-Quilos, verificada em Alagoas; c) Abrilada, detonada no Rio de Janeiro; d) Revolta dos Mals, ocorrida na Bahia; e) Revolta do "Maneta", destravada em Pernambuco.

07. (MACKENZIE) Marque a alternativa que completa corretamente o texto seguinte: "As causas da ___________ eram anunciadas por Bento Gonalves no manifesto de 29 de agosto de 1838, denunciando as altas tarifas sobre os produtos regionais: ouro, sebo, charque e graxa, poltica esta responsvel pela separao da provncia de So Pedro do Rio Grande do Sul da Comunidade Brasileira." a) Cabanagem b) Balaiada c) Farroupilha d) Sabinada e) Confederao do Equador 08. (UCSAL) Durante as primeiras dcadas do Imprio, a Bahia passou grande agitao poltica e social. Ocorreram vrias revoltas contra a permanncia de portugueses que haviam lutado contra os baianos na Guerra da Independncia. Entre as revoltas a que o texto se refere pode-se destacar, a: a) Farroupilha b) Praieira c) Balaiada d) Cabanagem e) Sabinada 09. (FUVEST) A Sabinada que agitou a Bahia entre novembro de 1837 e maro de 1838: a) tinha objetivos separatistas, no que diferia frontalmente das outras rebelies do perodo; b) foi uma rebelio contra o poder institudo no Rio de Janeiro que contou com a participao popular; c) assemelhou-se Guerra dos Farrapos, tanto pela posio anti-escravista quanto pela violncia e durao da luta; d) aproximou-se, em suas proposies polticas, das demais rebelies do perodo pela defesa do regime monrquico;

e) pode ser vista como uma continuidade da Rebelio dos Alfaiates, pois os dois movimentostinham os mesmos objetivos. 10. (UMC) O Golpe da Maioridade, datado de julho de 1840 e que elevou D. Pedro II a imperador do Brasil, foi justificado como sendo: a) uma estratgia para manter a unidade nacional, abalada pelas sucessivas rebelies provinciais; b) o nico caminho para que o pas alcanasse novo patamar de desenvolvimento econmico e social; c) a melhor sada para impedir que o Partido Liberal dominasse a poltica nacional; d) a forma mais vivel para o governo aceitar a proclamao da Repblica e a abolio da escravido; e) uma estratgia para impedir a instalao de um governo ditatorial e simpatizante do socialismo utpico.

Resoluo: 01. D 02. D 03. B 04. A 05. C 06. D 07. C 08. E 09. B 10. A

Biologia
Questes: 01. Quais so os quatro compostos orgnicos fundamentais encontrados nos seres vivos? 02. A histologia divide os tecidos animais e quatro grupos bsicos. Quais so eles? 03. Qual foi a estrutura celular observada por Robert Hooke? 04. Assinale a afirmao que no faz parte da teoria celular: a) Os seres vivos so formados por clulas. b) Os fenmenos fundamentais da vida ocorrem em nvel celular. c) Toda clula resulta da diviso ou fuso de clulas pr-existentes. d) Em todos os seres vivos as clulas realizam o mesmo tipo de ciclo celular. e) As clulas-me transmitem suas caractersticas s clulas-filhas. 05. So organismos procariontes: a) vrus e bactrias; b) vrus e cianofceas; c) bactrias e cianofceas; d) bactrias e fungos; e) todos os unicelulares. 06. (UF - PA) As clulas variam em tamanho, desde grandes algas unicelulares, vitelo de ovos de aves, at minsculas bactrias com dimenses da ordem de 0,0001 mm. Com relao s dimenses da grande maioria das clulas e de suas estruturas, podemos dizer que: a) a maior parte das clulas medida em mm, suas estruturas microscpicas, em nm, e as estruturas submicroscpicas, em ;

b) a grande maioria das clulas medida em , suas estruturas microscpicas, em nm, e as estruturas submicroscpicas em em mm; c) a grande maioria das clulas medida em e suas estruturas microscpicas, em nm; d) todas as clulas s podem ser medidas em ; e) s as clulas microscpicas so medidas em mm. 07. O material gentico das clulas : a) a glicose b) uma protena c) o colesterol d) o cido desoxirribonuclico. e) um aminocido 08. Na composio qumica de uma clula existem componentes orgnicos e inorgnicos. Quais so esses componentes? 09. Por que as clulas de intensa atividade possuem maior quantidade de gua do que as clulas de pequenaatividade metablica? 10. A taxa de gua varia em funo de trs fatores bsicos: atividade metablica, idade e espcie. Baseado nesses dados, a alternativa que apresenta um conjunto de maior taxa hdrica : a) corao, ancio, cogumelo; b) estmago, criana, abacateiro; c) msculo da perna, recm-nascido, medusa; d) ossos da perna, adulto, "orelha-de-pau"; e) pele, adolescente, coral.

Resoluo: 01. So: acares, lipdeos, protenas e cidos nuclicos. 02. Tecido epitelial, tecido conjuntivo, tecido muscular e tecido nervoso. 03. Hooke observou apenas paredes celulares de clulas mortas. 04. D 05. C 06. A 07. D 08. Carboidratos, lipdeos, protenas, cidos nuclicos, gua e sais minerais. 09. O metabolismo celular acionado por enzimas que s entram em atividade na presena de gua. 10. C

Questes: 01. Descreva o modelo de mosaico fludo proposto por Singer e Nicholson para a estrutura da membrana plasmtica. 02. Em relao s especialidades da membrana, conhecidas como microvilosidades, responda: a) No que consistem? b) Para que servem? c) Onde aparecem? 03. No esquema abaixo aparecem clulas epiteliais da mucosa 1 designadas por 2.

1 a) b) c) d) e) bucal intestinal intestinal gstrica gstrica

2 microvilosidades microvilosidades microfilamentos desmossomos clios

04. Em relao s clulas animais, denominam-se glicoclix: a) os microvilos das clulas que revestem o intestino delgado; b) todas as clulas acinosas de glndulas secretoras; c) o colo do espermatozide; d) a estrutura lipoprotica das membranas celulares; e) uma camada com protenas e carboidratos que recobre a membrana plasmtica. 05. Na figura abaixo, a estrutura apontada pela seta 1 representa uma formao:

a) importante para a movimentao celular; b) importante para aumentar a superfcie celular, facilitando a absoro de substncias do meio externo; c) denominada vescula pinocittica; d) importante para manter a aderncia entre uma clula e outra; e) que contm grande quantidade de enzimas.

06. A membrana plasmtica apresenta uma propriedade tpica: a permeabilidade seletiva. No que consiste essa propriedade? 07. (VUNESP) A membrana plasmtica que delimita a clula permite a passagem seletiva de substncias do meio externo para o meio interno da clula e vice-versa. O que se entende por transporte ativo e difuso facilitada? 08. Todas as clulas possuem uma membrana plasmtica, ou plasmalema, que separa o contedo protoplasmtico, ou meio intracelular, do meio ambiente. A existncia e integridade dessa estrutura so importantes, porque a membrana: a) regula as trocas entre a clula e o meio, s permitindo a passagem de molculas de fora para dentro da clula e impedindo a passagem em sentido inverso; b) possibilita clula manter a composio intracelular diversa do meio ambiente; c) impede a penetrao de substncias existentes em excesso no meio ambiente; d) torna desnecessrio o consumo energtico para captao de metablitos do meio externo; e) impede a sada de gua do citoplasma. 09. Sobre o mecanismo de transporte ativo, atravs da membrana celular, so feitas as seguintes afirmaes: I - Para que molculas sejam transportadas a partir de uma soluo mais concentrada para uma menos concentrada, atravs da membrana celular, a clula deve despender energia, e isto denominado transporte ativo. II - Dentre as diferentes substncias que so, com freqncia, transportadas ativamente atravs da membrana celular esto:aminocidos, ons sdio, ons potssio, ons hidrognio e vrios monossacardeos. III - O mecanismo bsico envolvido no transporte ativo depende de transportadores especficos, que reagem de maneira reversvel com as substncias transportadas, sob a ao de enzimas e com consumo de energia. Escreveu-se corretamente em: a) I e II apenas b) I e III apenas c) II e III apenas d) I, II e III e) nenhuma delas 10. O que acontece quando as hemcias so colocadas em: a) gua destilada. b) Soluo hipertnica.

Resoluo: 01. Dupla camada lipdica, na qual se movimentam molculas proticas. 02. a) Evaginaes da membrana plasmtica. b) Aumentar a superfcie de absoro. c) Clulas do epitlio intestinal. 03. B

04. E 05. D 06. Consiste em regular a entrada e sada de substncias na clula, mantendo uma composio qumica especfica. 07. O transporte ativo feito contra gradiente de construo e consumo de ATP (energia). A difuso facilitada feita sem ATP, com o auxlio de permeases, molculas proticas. 08. B 09. D 10. a) hemlise b) crenao (desidratao) Exerccios sobre meiose Leia o artigo: Mitose e Meiose

Questes: 01. (FUVEST) Considere os processos de mitose e meiose. a) Qual o nmero de cromossomos das clulas originadas, respectivamente, pelos doisprocessos, na espcie humana? b) Qual a importncia biolgica da meiose? 02. Em que tipos celulares acontece a meiose? 03.

a) Temos meiose em A, B e C. b) Temos mitose em A e meiose em B e C. c) Temos meiose em A e mitose em B e C. d) Temos meiose em A e B e mitose em C. e) Temos mitose em A e B e meiose em C. 04. O esquema ao lado apresenta um ciclo vital:

Os nmeros I, II e III representam, respectivamente: a) mitose, fecundao, meiose; b) meiose, fecundao, mitose; c) mitose, meiose, fecundao; d) meiose, mitose, fecundao; e) fecundao, mitose, meiose. 05. Uma clula com 8 cromossomos sofre meiose e origina: a) 2 clulas com 4 cromossomos; b) 2 clulas com 8 cromossomos; c) 4 clulas com 2 cromossomos; d) 4 clulas com 4 cromossomos; e) 4 clulas com 8 cromossomos; 06. Que fenmenos celulares ocorrem nas quatro fases da espermatognese? 07. O esquema abaixo mostra uma das etapas da ovognese:

Pergunta-se: a) Qual o nome da fase representada? b) Quais so as clulas haplides? c) Como se chamam, respectivamente, as clulas 1, 3 e 4? 08. Sabe-se que a seqncia da espermatognese a seguinte: espermatognia espermatcitos I espermatcitos II espermtides espermatozides. Pergunta-se: Quantos espermatozides sero produzidos, respectivamente, a partir de 100 espermtides e 100 espermatcitos I? a) 400 e 400 b) 400 e 100 c) 100 e 800 d) 200 e 400 e) 100 e 400

09. Analise as afirmativas abaixo, relacionadas com a gametognese: I - Todas as clulas que se originam do espermatcito I convertem-se em gametas. II - Todas as clulas que se originam do ovcito I convertem-se em gametas. III - Somente uma das quatro clulas que se originam do ovcito I converte-se em gameta. Assinale: a) se somente I e II forem corretas; b) se somente I e III forem corretas; c) se somente II e III forem corretas; d) se apenas uma delas for correta; e) se todas forem corretas. 10. A partir de cada ovognia obtm-se, geralmente: a) dois vulos e dois corpsculos polares; b) trs corpsculos polares e um vulo; c) trs vulos e um corpsculo polar; d) quatro corpsculos polares; e) quatro vulos.

Resoluo: 01. a) A mitose produz clulas com 46 cromossomos; a meiose clulas com 23. b) Manuteno do nmero cromossmico da espcie e aumento da variabilidade. 02. Clulas germinativas, clulas-me de esporos. 03. E 04. B 05. D 06. Mitose, interfase, meiose e diferenciao celular. 07. a) Maturao b) 2, 3, 4, 5 e 6 c) 1 - ovcito de primeira ordem 3 - vulo 4 - primeiro glbulo polar 08. E 09. B 10. B Exerccios sobre o citoplasma Leia o artigo: Organelas Citoplasmticas

Questes: 01. As clulas que sintetizam grandes quantidades de protena apresentam nuclolo bastantedesenvolvido. Por qu?

02. (UNICAMP) Uma droga qualquer de efeito txico (fenobarbital, por exemplo, que um sedativo usado como medicamento) foi fornecida a ratos adultos por 5 dias consecutivos. O retculo endoplasmtico (RE) de hepatcitos (clulas do fgado), analisado durante 12 dias, apresentou os seguintes resultados:

O que sugerem os resultados obtidos? Por qu? 03. (UF-PA) Sobre as funes dos tipos de retculo endoplasmtico, podese afirmar que: a) o rugoso est relacionado com o processo de sntese de esterides; b) o liso tem como funo a sntese de protenas; c) o liso responsvel pela formao do acrossomo dos espermatozides; d) o rugoso est ligado sntese de protena; e) o liso responsvel pela sntese de poliolosdios. 04. (UF-CE) O aspecto comum do Complexo de Golgi, em clulas animais, deduzindo atravs de observaes ao microscpio eletrnico, de: a) vesculas formadas por dupla membrana, sendo a interna sem granulaes e com dobras voltadas para o interior; b) membranas granulosas delimitando vesculas e sacos achatados, que dispem paralelamente; c) um complexo de membranas formando tubos anastomosados, com dilataes em forma de disco; d) sacos e vesculas achatadas, formadas por membrana dupla em que a interna, cheia de grnulos, emite para o interior prolongamentos em forma de dobras; e) membranas lisas delimitando vesculas e sacos achatados, que se dispem paralelamente. 05. (VUNESP) Numa clula eucaritica, a sntese de protenas, a sntese de esterides e a respirao celular esto relacionadas, respectivamente: a) ao Complexo de Golgi, s mitocndrias, aos ribossomos; b) ao retculo endoplasmtico liso, ao retculo endoplasmtico granular, ao Complexo de Golgi; c) aos ribossomos, ao retculo endoplasmtico liso, s mitocndrias; d) ao retculo endoplasmtico granular, s mitocndrias, ao Complexo de Golgi; e) ao retculo endoplasmtico liso, ao Complexo de Golgi, s mitocndrias. 06. Os lisossomos participam de dois processos celulares: autofagia e autlise. No que consiste esses dois processos? 07. De que maneira a clula age em relao gua oxigenada, produto txico resultante da atividade

celular? 08. (UF - So Carlos) Todas as alternativas abaixo expressam uma relao correta entre uma estrutura celular e sua funo ou origem, exceto: a) Aparelho de Golgi - relacionado com a sntese de polissacardeos e com a adio de acares s molculas de protenas. b) Retculo endoplasmtico rugoso - relacionado com a snteses de protenas reduzidas das clulas. c) Peroxissomos - relacionados com os processos de fagocitose e pinocitose, sendo responsveis pela digesto intracelular. d) Lisossomos - ricos em hidrolases cidas, tm sua origem relacionada com os sacos do aparelho de Golgi. e) Retculo endoplasmtico liso - relacionado com a secreo de esterides e com o processo de desintoxicao celular. 09. Qual das estruturas abaixo est associada corretamente principal funo que exerce na clula? a) centro celular ................................... diviso celular b) lisossomo ........................................ respirao celular c) mitocndria ..................................... sntese de protena d) ribossomo ....................................... digesto celular e) Complexo de Golgi ......................... ciclos de Krebs 10. Associe a segunda coluna de acordo com a primeira e assinale a alternativa que est correta: 1. Lisossomo 2. Complexo de Golgi 3. Ribossomo 4. Retculo endoplasmtico 5. Mitocndria ( ( ( ( ( ) Acmulo e eliminao de secreo ) Digesto intracelular ) Transporte de materiais dentro da clula ) Sntese de protenas ) Armazenamento de substncias

Resoluo: 01. Porque o nuclolo forma o RNA ribossmico, constituinte fundamental dos ribossomos. 02. Devido sua funo na destoxificao celular o RE sofre hipertrofia. 03. D 04. E 05. C 06. Autofagia a digesto de estruturas celulares em desuso; serve para a renovao celular. Autlise a desintegrao celular. 07. Os peroxissomos produzem a calatase, enzima que transforma a gua oxigenada em gua e oxignio. 08. C 09. A 10. B Exerccios sobre o ciclo celular - mitose Leia o artigo: Mitose e Meiose

Questes: 01. Indicar as fases da mitose em que ocorrem os fenmenos abaixo citados: a) aumento do volume nuclear b) condensao mxima dos cromossomos c) diviso dos centrmeros d) diviso do citoplasma e) migrao polar dos cromossomos 02. Em que fase da vida celular os cromossomos so mais facilmente visveis? Por qu? 03. No final da mitose, a reconstituio da carioteca (envoltrio nuclear) est mais intimamente relacionada com: a) Retculo endoplasmtico b) Complexo de Golgi c) Mitocndria d) Lisossomo e) Membrana citoplasmtica 04. O grfico abaixo representa a distncia entre pares de cromossomos homlogos durante a mitose. A anfase inicia-se aos:

a) 15 minutos b) 18 minutos c) 20 minutos d) 25 minutos e) 30 minutos 05. Os esquemas I, II e III representam diferentes estgios da mitose.

Com relao a estes estgios afirma-se: 1) O esquema III representa o estgio de anfase. 2) O esquema II representa o estgio de metfase. 3) durante o estgio representado pelo esquema II que ocorre o rompimento do envoltrio nuclear. 4) durante o estgio representado pelo esquema I que ocorre a separao das cromtides irms. 5) Durante a diviso celular, o estgio representado pelo esquema III precede o estgio representado pelo esquema I. 6) Durante a diviso celular, o estgio representado pelo esquema II precede o estgio representado pelo esquema III. Quais so as afirmaes corretas: a) apenas 3, 4, 5 e 6 b) todas c) apenas 1, 2, 3, e 5 d) nenhuma e) apenas 1, 3, 5 e 6 06. A interfase um perodo em que as clulas esto em repouso. Voc concorda? Justifique sua resposta. 07. A interfase dividida em trs perodos: G1, S e G2. O que acontece em cada um deles? 08. (UFPB) A quantidade de DNA de uma clula somtica em metfase mittica X. Clulas do mesmo tecido, mas nas fases G1 e G2, devem apresentar uma quantidade de DNA, respectivamente, igual a: a) X e X 2 b) X e X 2 c) 2X e X d) X e 2X e) X e 2X 09. Qual das seguintes estruturas no est presente em uma clula de raiz de cebola que sofre mitose? a) parede celular b) centrolo c) centrmero d) mitocndria e) fuso 10. Por que a mitose da clula vegetal chamada de acntrica e anfase?

Resoluo: 01. a) prfase d) telfase 03. A 04. A b) metfase e) anfase c) anfase

02. Metfase, devido ao grau mximo de condensao.

05. A 06. No, a atividade metablica intensa, com sntese de DNA, RNA e protenas. 07. G1: intensa sntese de RNA e protenas. S: sntese de DNA. G2: reduzida sntese de RNA e protenas. 08. A 09. B 10. Por no apresentar centrolo e ster. Exerccios sobre transporte de nutrientes Leia o artigo: Reino Plantae

Questes: 01. O que ocorre com o transporte da seiva mineral nos vegetais, segundo a Teoria de Dixon, quando as folhas das rvores caem no inverno? 02. Quais so os tipos celulares que constituem o xilema? 03. Qual a funo dos reforos de lignina, observados ao longo dos elementos dos vasos e das traquedes? 04. Um ramo vegetal seccionado foi mergulhado em uma soluo de corante. Aps algum tempo as nervuras foliares ficaram coloridas. Qual o nome do tecido que transportou o corante at as folhas? 05. (UFPE) Complete a frase: Quando uma planta transpira intensamente, a seiva bruta circula ______________ e o colapso dos vasos evitado devido presena de ___________________. a) em estado de tenso vlvulas dispostas ao longo dos vasos b) com presso positiva depsitos de calose nos vasos lenhosos c) com presso negativa depsitos de suberina nas placas crivadas d) em estado de tenso reforos de lignina e) com presso positiva absoro de ons minerais 06. Numa planta que transpira intensamente, a seiva bruta nos vasos lenhosos acha-se: a) conduzida apenas pelos microcapilares da membrana celulsica; b) conduzida pelo xilema com presso positiva; c) conduzida pelo xilema em estado de tenso; d) conduzida com presso positiva ou negativa, dependendo do tamanho da planta; e) conduzida pelo lber com presso positiva. 07. Suponha que em certas horas iluminadas de um dia de vero todos os estmatos de uma planta estejam fechados. Qual dos pares de atividades abaixo mencionados ser o menos prejudicado nessa situao? a) Fotossntese e transporte de gua pelo xilema. b) Transporte de gua pelo xilema e absoro pelas razes. c) Respirao e transpirao. d) Fotossntese e respirao.

e) Transporte e absoro de gua pelas razes. 08. (UNICAMP) Estima-se que uma nica planta de milho, com 0,4 kg de peso, absorve 130 a 180 litros de gua ao longo de sua vida. Sabendo-se que apenas cerca de 2% de toda gua absorvida utilizada na fotossntese e em outras atividades metablicas, qual o destino do excedente de gua? Indique a trajetria da gua na planta e as estruturas envolvidas. 09. Em plantas mantidas em ambiente saturado de vapor dgua, cessa: a) a produo de auxinas; b) a sntese de matria orgnica; c) a difuso de CO2 pelos estmatos; d) o transporte de seiva elaborada; e) o transporte de gua pelo xilema. 10. O floema: a) Est constitudo, principalmente, pelos vasos denominados liberianos, formados por clulas mortas, impregnadas por lignina. b) Transporta a gua e os sais minerais, absorvidos do solo, para todas as partes da planta. c) So estruturas vasculares altamente desenvolvidas nas brifitas. d) o responsvel pelo transporte de gua e substncias orgnicas formadas nas folhas. e) Est constitudo por vrios elementos, sendo os vasos lenhosos os mais importantes.

Resoluo: 01. Manter as paredes celulares rgidas, impedindo o colapso dos vasos lenhosos. 02. a) Sistema traquerio (elementos do vaso e traquedes) b) Parnquima lenhoso c) esclernquima 03. Manter as paredes celulares rgidas impedindo o colapso dos vasos lenhosos.Cessa omovimento de seiva bruta. 04. Lenho ou xilema 05. D 06. C 07. D 08. A maior parte da gua perdida por transpirao Trajetria da gua: Plos absorventes crtex da raiz endoderma periciclo xilema parnquimas foliares estmatos. 09. E 10. D Exerccios sobre osmose e absoro Leia o artigo: Osmose

Questes: 01. (PUC SP) O esquema abaixo mostra o comportamento da clula vegetal submetida a duas condies osmticas diferentes:

a) Como so denominadas as clulas A e B? b) Considere Sc = suco celular; Si = suco vacuolar e M = resistncia da membrana celulsica. Se, no caso da clula A, o valor de Si de 10 atmosferas, qual o valor esperado, em atmosferas, de Sc e M para essa clula? 02. (FUVEST) Sabendo-se que o dficit de presso de difuso de uma clula (DPD) a diferena entre a presso osmtica (PO) e a presso de turgor (PT), qual das seguintes alternativas representa a situao de uma clula trgida? a) DPD = PT b) DPD = PO c) PT = 1 d) PO = 1 e) DPD = 0 03. (VUNESP) Um estudante colocou dois pedaos recm-cortados de um tecido vegetal em dois recipientes, I e II, contendo soluo salina. Depois de algumas horas, verificou que no recipiente I as clulas do tecido vegetal estavam plasmolisadas. No recipiente II, as clulas mantiveram o tamanho normal. Qual a concluso do estudante quanto: a) a concentrao das solues salinas nos recipientes I e II, em relao ao suco celular desse tecido? b) O que significa dizer que em I as clulas estavam plasmolisadas? 04. (FEI) Uma clula vegetal plasmolisada poder voltar situao normal desde que seja colocada em uma soluo: a) hipotnica, em relao concentrao do seu suco celular; b) hipertnica, em relao concentrao de seu vacolo; c) isotnica, em relao concentrao de seu suco celular; d) de concentrao maior do que a existente no seu interior; e) de concentrao igual existente no suco vacuolar. 05. (UERJ) O esquema abaixo demonstra o fenmeno da osmose. No incio do experimento, h uma soluo de glicose a 50% dentro de um recipiente envolvido por uma membrana que impermevel glicose mas no gua.

A variao da taxa osmtica em funo do tempo, no sistema envolvido por membrana, est apresentada precisamente pelo seguinte grfico:

a)

b)

c)

d)

e)

06. (FUVEST) Clulas vegetais, como as representadas na figura A, foram colocadas em uma determinada soluo e, no fim do experimento, tinham aspecto semelhante ao da figura B. Comparando-se as concentraes do interior da clula na situao inicial (I), da soluo externa (II) e do interior da clula na situao final (III), podemos dizer que:

a) I maior que II b) I maior que III c) I menor que II d) I igual a III e) III maior que II 07. (UFPA) O comportamento de hemcias e clulas vegetais, quando colocadas em meios hipotnicos, diferente em virtude da:

a) diferena de funcionamento da membrana plasmtica; b) existncia de vacolos nas clulas vegetais; c) existncia de parede celulsica nas clulas vegetais; d) porosidade da membrana celulsica; e) inexistncia de ncleo das hemcias. 08. (UEL) Numa clula vegetal encontrou-se um valor de 25 atm para presso osmtica do vacolo, sendo que igual valor foi encontrado para presso de turgescncia. Conclui-se que a clula est: a) murcha b) trgida c) plasmolisada d) em plasmlise incipiente e) com turgor negativo 09. (UFMG) O esquema representa uma clula turgescente de parnquima aclorofilado. O comprimento das setas, que indicam movimento de substncia, proporcional s quantidades de substncias que fluem:

Satisfazem as situaes I, II e III do esquema, as seguintes substncias, respectivamente: a) glicose, gua e amido b) glicose, amido e gua c) gua, glicose e amido d) gua, amido e glicose e) amido, glicose e gua 10. (UEMT) O diagrama abaixo representa as variaes de volume em clulas vegetais em relao s condies hdricas:

As clulas na posio A esto _______; para atingir o ponto A foram mergulhadas em _______. a) murchas meio hipertnico b) plasmolisadas meio hipertnico c) murchas meio isotnico d) trgidas meio hipotnico

e) trgidas meio isotnico

Resoluo: 01. a) A Clula trgida B Clula plasmolizada b) Frmula: Sc = Si - M para Si = 10 atm tem-se: Sc = 0 M = 10 porque na clula trgida Si = M e Sc = 0 02. B 03. a) I Soluo hipertnica II Soluo isotnica b) A clula perdeu gua e o citoplasma descolou-se, afastando-se da parede celular. 04. C 08. B 05. B 09. A 06. C 10. D 07. C

Matemtica
Exerccios sobre equao algbrica Leia o artigo: lgebra

Questes: 01. (VUNESP) Assinale a alternativa que indica o polinmio que possui os nmeros 0 e 1 como razes, sendo 0 uma raiz de multiplicidade 3: a) p(x) = x (x3 - 1) b) p(x) = x (x - 1)3 c) p(x) = x3 (x - 1) d) p(x) = (x3 - x) (x - 1) e) p(x) = x (x3 + x2 - 2) 02. (PUCCAMP) Sabe-se que a equao 2x3 + x2 - 6x - 3 = 0 admite uma nica raiz racional e no inteira. As demais razes dessa equao so: a) inteiras e positivas; b) inteiras e de sinais contrrios; c) no reais; d) irracionais e positivas; e) irracionais e de sinais contrrios. 03. O polinmio de coeficientes inteiros, de menor grau possvel, que tem como razes 2 e i, pode ser: a) x3 - 2x2 - x + 2 b) x2 + (2 - i) x - 2 c) x2 - (2 + i) x + 2i d) x3 - 2x2 + x - 2

e) x3 + x2 - x - 2 04. (FUVEST) A equao x3 + mx2 + 2x + n = 0, em que m e n so nmeros reais, admite 1 + i (i sendo a unidade imaginria) como a raiz. Ento m e n valem, respectivamente: a) 2 e 2 b) 2 e 0 c) 0 e 2 d) 2 e -2 e) -2 e 0 05. Sabe-se que o nmero complexo i soluo da equao x4 - 3x2 - 4 = 0. Ento: a) essa equao tem uma soluo de multiplicidade 2; b) as solues dessa equao formam uma progresso; c) a equao tem duas solues reais irracionais; d) a equao tem 2 solues reais racionais; e) a equao no tem solues reais. 06. Determinar a sabendo-se que 2 raiz da equao x4 - 3x3 + 2x2 + ax - 3 = 0. 07. Resolver a equao x4 - 5x2 - 10x - 6 = 0, sabendo-se que duas de suas razes so -1 e 3. 08. Resolver a equao x3 - 3x2 - x + 3 = 0, sabendo-se que a soma de duas razes zero. 09. Sabendo-se que 1 a raiz da equao x3 - 2x2 + ax + 6 = 0, determinar a e as demais razes da equao. 10. Sendo P(x) um polinmio de 5 grau que satisfaz as condies 1 = P(1) = P(2) = P(3) = P(4) = P(5) = P(6) = 0, obter o conjunto-verdade da equao P(x) - 1 = 0 e o valor de P(0).

Resoluo: 01. C 02. E 03. D 04. E 05. D 06. a = 3/2 07. V = {-1; 3; -1 + 1; -1 - i} 08. O conjunto-verdade da equao {-1; 1; 3} 09. a = -5 e as demais razes so -2 e 3. 10. V = {1; 2; 3; 4; 5} e P(0) = 2 Exerccios sobre fatorao Leia o artigo: Equaes

Questes: 01. Fatorar: (a + b) . x + 2(a + b) 02. Fatorar: (x + y)2 - (x - y)2 03. Fatorar: x4 - y4 04. Fatorar: 25x2 + 70x + 49 05. Calcular 2 4992 06. Dado que x = a + x-1, a expresso x2 + x-2 igual a: a) a2 + 2 b) 2a + 1 c) a2 + 1 d) 2a -1 e) a2 07. (PUC) Sendo x3 + 1 = (x + 1) (x2 + ax + b) para todo x real, os valores de a e b so, respectivamente: a) -1 e -1 b) 0 e 0 c) 1 e 1 d) 1 e -1 e) -1 e 1 08. Decomponha em fatores do primeiro grau: 6x2 - 5xy + y2 09. (FUVEST) A soma dos quadrados de dois nmeros positivos 4 e a soma dos inversos de seus quadrados 1. Determine: a) O produto dos dois nmeros. b) A soma dos dois nmeros. 10. (FUVEST) A diferena entre o cubo da soma de dois nmeros inteiros e a soma de seus cubos pode ser: a) 4 b) 5 c) 6 d) 7 e) 8

Resoluo: 01. (a + b) . (x . 2)

02. 4xy 03. (x2 + y2) . (x + y) . (x - y) 04. (5x + 7)2 05. 6 245 001 06. A 07. E

08. (3x - y) . (2x - y) 09. a) 2

10. C Exerccios sobre razo e proporo Leia o artigo: Proporo urea

Questes: 01. Se (3, x, 14, ...) e (6, 8, y, ...) forem grandezas diretamente proporcionais, ento o valor de x + y : a) 20 b) 22 c) 24 d) 28 e) 32 02. Calcular x e y sabendo-se que (1, 2, x, ...) e (12, y, 4, ...) so grandezas inversamente proporcionais. 03. Dividir o nmero 160 em trs partes diretamente proporcionais aos nmeros 2, 3 e 5. 04. Repartir uma herana de R$ 495.000,00 entre trs pessoas na razo direta do nmero defilhos e na razo inversa das idades de cada uma delas. Sabe-se que a 1 pessoa tem 30 anos e 2 filhos, a 2 pessoa tem 36 anos e 3 filhos e a 3 pessoa 48 anos e 6 filhos. 05. Dois nmeros esto na razo de 2 para 3. Acrescentando-se 2 a cada um, as somas esto na razo de 3 para 5. Ento, o produto dos dois nmeros : a) 90 b) 96 c) 180 d) 72 e) -124 06. (PUC) Se (2; 3; x; ...) e (8; y; 4; ...) forem duas sucesses de nmeros diretamente proporcionais, ento: a) x = 1 e y = 6 b) x = 2 e y = 12 c) x = 1 e y = 12 d) x = 4 e y = 2

e) x = 8 e y = 12 07. Sabe-se que y diretamente proporcional a x e que y = 10 quando x = 5. De acordo com estes dados, qual: a) a sentena que relaciona y com x? b) o grfico da funo f: [-2; 3] definida pela sentena anterior? c) o valor de y quando x = 2? 08. (FUVEST) So dados trs nmeros reais, a < b < c. Sabe-se que o maior deles a soma dos outros dois e o menor um quarto do maior. Ento a, b e c so, respectivamente, proporcionais a: a) 1, 2 e 3 b) 1, 2 e 5 c) 1, 3 e 4 d) 1, 3 e 6 e) 1, 5 e 12 09. (MACK) Dividindo-se 70 em partes proporcionais a 2, 3 e 5, a soma entre a menor e a maior parte : a) 35 b) 49 c) 56 d) 42 e) 28 10. (UFLA) Trs pessoas montam uma sociedade, na qual cada uma delas aplica, respectivamente, R$ 20.000,00, R$ 30.000,00 e R$ 50.000,00. O balano anual da firma acusou um lucro de R$ 40.000,00. Supondo-se que o lucro seja dividido em partes diretamente proporcionais ao capital aplicado, cada scio receber, respectivamente: a) R$ 5.000,00; R$ 10.000,00 e R$ 25.000,00 b) R$ 7.000,00; R$ 11.000,00 e R$ 22.000,00 c) R$ 8.000,00; R$ 12.000,00 e R$ 20.000,00 d) R$ 10.000,00; R$ 10.000,00 e R$ 20.000,00 e) R$ 12.000,00; R$ 13.000,00 e R$ 15.000,00

Resoluo: 01. E 02. x = 3 e y = 6 03. As partes so: 32, 48 e 80. 04. A 1 pessoa deve receber R$ 120.000,00, a 2 pessoa R$ 150.000,00 e a terceira pessoa R$ 225.000,00. 05. B 06. C 07. a) y = 2x

c) y = 4 08. C 09. B 10. Cvg

Portugus
Exerccios sobre figuras de linguagem Leia o artigo: Figuras de Linguagem

Questes: 01. (VUNESP) No trecho: "...do um jeito de mudar o mnimo para continuar mandando o mximo", a figura de linguagem presente chamada: a) metfora b) hiprbole c) hiprbato d) anfora e) anttese 02. (PUC - SP) Nos trechos: "O pavo um arco-ris de plumas" e "...de tudo que ele suscita e esplende e estremece e delira..." enquanto procedimento estilstico, temos, respectivamente: a) metfora e polissndeto; b) comparao e repetio; c) metonmia e aliterao; d) hiprbole e metfora; e) anfora e metfora. 03. (PUC - SP) Nos trechos: "...nem um dos autores nacionais ou nacionalizados de oitenta pra l faltava nas estantes do major" e "...o essencial achar-se as palavras que o violo pede e deseja" encontramos, respectivamente, as seguintes figuras de linguagem: a) prosopopia e hiprbole; b) hiprbole e metonmia; c) perfrase e hiprbole; d) metonmia e eufemismo; e) metonmia e prosopopia. 04. (VUNESP) Na frase: "O pessoal esto exagerando, me disse ontem um camel", encontramos a

figura de linguagem chamada: a) silepse de pessoa b) elipse c) anacoluto d) hiprbole e) silepse de nmero 05. (ITA) Em qual das opes h erro de identificao das figuras? a) "Um dia hei de ir embora / Adormecer no derradeiro sono." (eufemismo) b) "A neblina, roando o cho, cicia, em prece. (prosopopia) c) J no so to freqentes os passeios noturnos na violenta Rio de Janeiro. (silepse de nmero) d) "E fria, fluente, frouxa claridade / Flutua..." (aliterao) e) "Oh sonora audio colorida do aroma." (sinestesia) 06. (UM - SP) Indique a alternativa em que haja uma concordncia realizada por silepse: a) Os irmos de Teresa, os pais de Jlio e ns, habitantes desta pacata regio, precisaremos de muita fora para sobreviver. b) Podero existir inmeros problemas conosco devido s opinies dadas neste relatrio. c) Os adultos somos bem mais prudentes que os jovens no combate s dificuldades. d) Dar-lhe-emos novas oportunidades de trabalho para que voc obtenha resultados mais satisfatrios. e) Haveremos de conseguir os medicamentos necessrios para a cura desse vrus insubordinvel a qualquer tratamento. 07. (FEI) Assinalar a alternativa correta, correspondente figuras de linguagem, presentes nos fragmentos abaixo: I. "No te esqueas daquele amor ardente que j nos olhos meus to puro viste." II. "A moral legisla para o homem; o direito para o cidado." III. "A maioria concordava nos pontos essenciais; nos pormenores porm, discordavam." IV. "Isaac a vinte passos, divisando o vulto de um, pra, ergues a mo em viseira, firma os olhos." a) anacoluto, hiprbato, hiplage, pleonasmo; b) hiprbato, zeugma, silepse, assndeto; c) anfora, polissndeto, elipse, hiprbato; d) pleonasmo, anacoluto, catacrese, eufemismo; e) hiplage, silepse, polissndeto, zeugma. 08. (FEBA - SP) Assinale a alternativa em que ocorre aliterao: a) "gua de fonte .......... gua de oceano ............. gua de pranto. (Manuel Bandeira) b) "A gente almoa e se coa e se roa e s se vicia." (Chico Buarque) c) "Ouo o tique-taque do relgio: apresso-me ento." (Clarice Lispector) d) "Minha vida uma colcha de retalhos, todos da mesma cor." (Mrio Quintana) e) N.d.a. 09. (CESGRANRIO) Na frase "O fio da idia cresceu, engrossou e partiu-se" ocorre processo de gradao. No h gradao em: a) O carro arrancou, ganhou velocidade e capotou.

b) O avio decolou, ganhou altura e caiu. c) O balo inflou, comeou a subir e apagou. d) A inspirao surgiu, tomou conta de sua mente e frustrou-se. e) Joo pegou de um livro, ouviu um disco e saiu. 10. (FATEC) "Seus culos eram imperiosos." Assinale a alternativa em que aparece a mesma figura de linguagem que h na frase acima: a) "As cidades vinham surgindo na ponte dos nomes." b) "Nasci na sala do 3 ano." c) "O bonde passa cheio de pernas." d) "O meu amor, paralisado, pula." e) "No serei o poeta de um mundo caduco."

Resoluo: 01. E 02. A 03. E 04. E 05. C 06. C 07. B 08. B 09. E 10.C Exerccios sobre funes de linguagem Leia o artigo: Funes da Linguagem e Comunicao Questes: 01. Reconhea nos textos a seguir, as funes da linguagem: a) "O risco maior que as instituies republicanas hoje correm no o de se romperem, ou serem rompidas, mas o de no funcionarem e de desmoralizarem de vez, paralisadas pela sem-vergonhice, pelo hbito covarde de acomodao e da complacncia. Diante do povo, diante do mundo e diante de ns mesmos, o que preciso agora fazer funcionar corajosamente as instituies para lhes devolver a credibilidade desgastada. O que preciso (e j no h como voltar atrs sem avacalhar e emporcalhar ainda mais o conceito que o Brasil faz de si mesmo) apurar tudo o que houver a ser apurado, doa a quem doer." (O Estado de So Paulo) b) O verbo infinitivo Ser criado, gerar-se, transformar O amor em carne e a carne em amor; nascer Respirar, e chorar, e adormecer E se nutrir para poder chorar Para poder nutrir-se; e despertar Um dia luz e ver, ao mundo e ouvir E comear a amar e ento ouvir E ento sorrir para poder chorar. E crescer, e saber, e ser, e haver E perder, e sofrer, e ter horror De ser e amar, e se sentir maldito E esquecer tudo ao vir um novo amor E viver esse amor at morrer E ir conjugar o verbo no infinito... (Vincius de Morais) c) "Para fins de linguagem a humanidade se serve, desde os tempos pr-histricos, de sons a que se d o nome genrico de voz, determinados pela corrente de ar expelida dos pulmes no fenmeno vital da respirao, quando, de uma ou outra maneira, modificada no seu trajeto at a parte exterior da boca."

(Matoso Cmara Jr.) d) " - Que coisa, n? - . Puxa vida! - Ora, droga! - Bolas! - Que troo! - Coisa de louco! - !" e) "Fique afinado com seu tempo. Mude para Col. Ultra Lights." f) "Sentia um medo horrvel e ao mesmo tempo desejava que um grito me anunciasse qualquer acontecimento extraordinrio. Aquele silncio, aqueles rumores comuns, espantavam-me. Seria tudo iluso? Findei a tarefa, ergui-me, desci os degraus e fui espalhar no quintal os fios da gravata. Seria tudo iluso?... Estava doente, ia piorar, e isto me alegrava. Deitar-me, dormir, o pensamento embaralhar-se longe daquelas porcarias. Senti uma sede horrvel... Quis ver-me no espelho. Tive preguia, fiquei pregado janela, olhando as pernas dos transeuntes." (Graciliano Ramos) g) " - Que quer dizer pitosga? - Pitosga significa mope. - E o que mope? - Mope o que v pouco." 02. No texto abaixo, identifique as funes da linguagem: "Gastei trinta dias para ir do Rossio Grande ao corao de Marcela, no j cavalgando o corcel do cego desejo, mas o asno da pacincia, a um tempo manhoso e teimoso. Que, em verdade, h dois meios de granjear a vontade das mulheres: o violento, como o touro da Europa, e o insinuativo, como o cisne de Leda e a chuva de ouro de Dnae, trs inventos do padre Zeus, que, por estarem fora de moda, a ficam trocados no cavalo e no asno." (Machado de Assis) 03. Descubra, nos textos a seguir, as funes de linguagem: a) "O homem letrado e a criana eletrnica no mais tm linguagem comum." (Rose-Marie Muraro) b) "O discurso comporta duas partes, pois necessariamente importa indicar o assunto de que se trata, e em seguida a demonstrao. (...) A primeira destas operaes a exposio; a segunda, a prova." (Aristteles) c) "Amigo Americano um filme que conta a histria de um casal que vive feliz com o seu filhoat o dia em que o marido suspeita estar sofrendo de cncer." d) "Se um dia voc for embora Ria se teu corao pedir Chore se teu corao mandar." (Danilo Caymmi & Ana Terra) e) "Ol, como vai? Eu vou indo e voc, tudo bem? Tudo bem, eu vou indo em pegar um lugar no futuro e voc? Tudo bem, eu vou indo em busca de um sono tranqilo..." (Paulinho da Viola) Texto para as questes 04 e 05 Potica Que poesia? uma ilha

cercada de palavras por todos os lados Que um poeta? um homem que trabalha um poema com o suor do seu rosto Um homem que tem fome como qualquer outro homem. (Cassiano Ricardo) 04. Quais as funes da linguagem predominantes no poema anterior? 05. Aponte os elementos que integram o processo de comunicao em Potica, de Cassiano Ricardo. 06. Historinha I

Historinha II

Qual a funo da linguagem comum s duas historinhas? 07. (CESUPA - CESAM - COPERVES) Segundo o lingsta Roman Jakobson, "dificilmente lograramos (...) encontrar mensagens verbais que preenchem uma nica funo... A estrutura verbal de uma mensagem depende basicamente da funo predominante". "Meu canto de morte Guerreiros, ouvi. Sou filho das selvas Nas selvas cresci. Guerreiros, descendo Da tribo tupi. Da tribo pujante, Que agora anda errante Por fado inconstante. Guerreiros, nasci: Sou bravo, forte, Sou filho do Norte Meu canto de morte, Guerreiros, ouvi."

(Gonalves Dias) Indique a funo predominante no fragmento acima transcrito, justificando a indicao. 08. (PUC - SP) "Com esta histria eu vou me sensibilizar, e bem sei que cada dia um dia roubado da morte. Eu no sou um intelectual, escrevo com o corpo. E o que escrevo uma nvoa mida. As palavras so sons transfundidos de sombras que se entrecruzam desiguais, estalactites, renda, msica transfigurada de rgo. Mal ouso clamar palavras a essa rede vibrante e rica, mrbida e obscura tendo como contratom o baixo grosso da dor. Alegro com brio. Tentarei tirar ouro do carvo. Sei que estou adiando a histria e que brinco de bola sem bola. O fato um ato? Juro que este livro feito sem palavras. uma fotografia muda. Este livro um silncio. Este livro uma pergunta." (Clarice Lispector) A obra de Clarice Lispector, alm de se apresentar introspectiva, marcada pela sondagem de fluxo de conscincia (monlogo interior), reflete, tambm, uma preocupao com a escritura do texto literrio. Observe o trecho em questo e aponte os elementos que comprovam tal preocupao. 09. (FATEC) O seno do livro COMEO a arrepender-me deste livro. No que ele me canse; eu no tenho que fazer; e, realmente, expedir alguns magros captulos para esse mundo sempre tarefa que distrai um pouco da eternidade. mas o livro enfadonho, cheira a sepulcro, traz certa contrao cadavrica, vcio grave, e alis nfimo, porque o maior defeito deste livro s tu, leitor. Tu tens pressa de envelhecer, e o livro anda devagar; tu amas a narrao direta e nutrida, o estilo regular e fluente, e este livro e o meu estilo so como os brios, guinam direita e esquerda, andam e param, resmungam, urram, gargalham, ameaam o cu, escorregam e caem... Este trecho revela o estilo de: a) MANUEL ANTONIO DE ALMEIDA, ao usar uma linguagem apelativa, direcionada reflexo crtica da obra romntica. b) GRACILIANO RAMOS, ao revelar a quebra da ordem cronolgica da narrativa de suas obras, como reflexo coerente da instabilidade psicolgica e espacial de suas personagens. c) MACHADO DE ASSIS, ao questionar o leitor quanto linha lgica e impositiva do tempo velho da obra literria e, ao mesmo tempo, conscientiz-lo de um novo modo de ler. d) LIMA BARRETO, ao retratar o estilo incoerente de suas personagens em seus atos de loucura. e) CARLOS DRUMMOND DE ANDRADE, ao especular o tempo e a qualidade de vida do homem (leitor) em interao com o tempo da narrativa.

Resoluo: 01. a) funo referencial c) funes referencial e metalingstica e) funo conativa g) funo metalingstica 02. Funo emotiva 03. a) funo referencial b) funo referencial c) funes referencial e metalingstica b) funo potica d) funo ftica f) funo emotiva

d) funo potica e) funo ftica 04. Funes potica e metalingstica. 05. Cdigo, emissor e mensagem. 06. Funo metalingstica, ltimo quadro de cada historinha. 07. Funo emotiva - predominncia de 1 pessoa. 08. Nesse fragmento de Clarice Lispector, alm da preocupao introspectiva em fisgar elementos interiores, profundos, beirando uma revelao epifnica transcendental, h tambm a preocupao constante com a prpria escritura do texto literrio, usando-se a funo metalingstica. A discusso ou abordagem da tessitura narrativa aparece em passagens como: "As palavras so sons transfundidos de sombras que se entrecruzam desiguais, estalactites, renda, msica transfigurada de rgo. Mal ouso clamar palavras a essa rede vibrante e rica (...)", "Sei que estou adiando a histria e que brinco de bola sem bola. O fato um ato? Juro que este livro feito sem palavras (...)" e "Eu no sou um intelectual, escrevo com o corpo. E o que escrevo uma nvoa mida". 09. C Exerccios sobre as classes gramaticais Leia o artigo: Gramtica

Questes: 01. A frase em que os vocbulos sublinhados pertencem mesma classe gramatical, exercem a mesma funo sinttica e tm significado diferente : a) Curta o curta: aproveite o feriado para assistir ao festival de curta-metragem. b) O novo novo: ser que tudo j foi feito antes? c) O carro popular a 12.000 reais est longe de ser popular. d) trgico verificar que, na televiso brasileira, s o trgico que faz sucesso. e) O Brasil ser um grande parceiro e no apenas um parceiro grande. 02. Um dos traos marcantes do atual perodo histrico (...) o papel verdadeiramente desptico da informao. (...) As novas condies tcnicas deveriam permitir a ampliao do conhecimento do planeta, dos objetos que o formam, das sociedades que o habitam e dos homens em sua realidade intrnseca. Todavia, nas condies atuais, as tcnicas da informao so principalmente utilizadas por um punhado de atores em funo de seus objetivos particulares. Essas tcnicas da informao (por enquanto) so apropriadas por alguns Estados e por algumas empresas, aprofundando assim os processos de criao de desigualdades. desse modo que a periferia do sistema capitalista acaba se tornando ainda mais perifrica, seja porque no dispe totalmente dos novos meios de produo, seja porque lhe escapa a possibilidade de controle. O que transmitido maioria da humanidade , de fato, uma informao manipulada que, em lugar de esclarecer, confunde. (Milton Santos, Por uma outra globalizao) Observe os sinnimos indicados entre parnteses: I o papel verdadeiramente desptico (= tirnico) da informao; II dos homens em sua realidade intrnseca (= inerente); III so apropriadas (= adequadas) por alguns Estados. Considerando-se o texto, a equivalncia sinonmica est correta APENAS em: a) I

b) II c) III d) I e II e) I e III 03. (FUVEST) Assinale a alternativa em que est correta a forma plural: a) jnior jniors b) mal maus c) fuzil fuzveis d) gavio gavies e) atlas atlas 04. Em que alternativa aparecem dois substantivos do gnero masculino? a) cal, dinamite b) lana-perfume, champanha c) alface, telefonema d) gengibre, omoplata e) formicida, sentinela 05. (FUVEST) Alm de parecer no ter rotao, a Terra parece tambm estar imvel no meio dos cus. Ptolomeu d argumentos astronmicos para tentar mostrar isso. Para entender esses argumentos, necessrio lembrar que, na antiguidade, imaginava-se que todas as estrelas (mas no os planetas) estavam distribudas sobre uma superfcie esfrica, cujo raio no parecia muito superior distncia da Terra aos planetas. Suponhamos agora que a Terra esteja no centro da esfera das estrelas. Neste caso, o cu visvel noite deve abranger, de cada vez, exatamente a metade da esfera das estrelas. E assim parece realmente ocorrer: em qualquer noite, de horizonte a horizonte, possvel contemplar, a cada instante, a metade do zodaco. Se, no entanto, a Terra estivesse longe do centro da esfera estelar, ento o campo de viso noite no seria, em geral, a metade da esfera: poderamos ver mais da metade, outras vezes poderamos ver menos da metade, de horizonte a horizonte. Portanto, a evidncia astronmica parece indicar que a Terra est no centro da esfera de estrelas. E se ela est sempre no centro, ela no se move em relao s estrelas. (Roberto de A. Martins, Introduo geral ao Commentariolus de Nicolau Copernico) Os termos alm de, no entanto, ento, portanto estabelecem, no texto, relaes, respectivamente, de: a) distanciamento objeo tempo efeito b) adio objeo tempo concluso c) distanciamento conseqncia concluso efeito d) distanciamento oposio tempo conseqncia e) adio oposio conseqncia concluso 6. Considerando a relao lgica existente entre os dois segmentos dos provrbios adiante citados, o espao pontilhado NO poder ser corretamente preenchido pela conjuno mas, apenas em: a) Morre o homem, (...) fica a fama. b) Reino com novo rei, (...) povo com nova lei. c) Por fora bela viola, (...) por dentro po bolorento. d) Amigos, amigos! (...) negcios parte. e) A palavra de prata, (...) o silncio de ouro. 7. (ESPM-SP) Preencha os espaos com sesso, seo, seco ou cesso.

Durante a ____________ parlamentar, uma ____________ do partido do Governo se manifestou contrria _____________ de terra a imigrantes do Japo. Texto para a questo 8 S os roados da morte compensam aqui cultivar, e cultiv-los fcil: simples questo de plantar; no se precisa de limpa, de adubar nem de regar; as estiagens e as pragas fazem-nos mais prosperar; e do lucro imediato; nem preciso esperar pela colheita: recebe-se na hora mesma de semear. (Joo Cabral de Melo Neto, Morte e vida severina) 8. Substituindo-se os dois-pontos por uma conjuno, em (...) pela colheita: recebe-se (...), mantm-se o sentido do texto APENAS em (...) pela colheita, a) embora de receba (...) b) ou se recebe (...) c) ainda que se receba (...) d) j que se recebe (...) e) portanto se recebe (...) 9. As frases a seguir esto dispostas aos pares. Leia-as com ateno e assinale a alternativa em que haja erro no emprego das palavras ou expresses destacadas. I. Pedro mora naquela casa h cerca de dois anos. Falamos bastante acerca de prostituio infantil II. Aquele aluno falava demais durante as aulas. Necessito de mais pacincia para lidar com eles. III. Preciso sair agora, se no perderei o nibus. Seno me ajudares, terei que recorrer a outra pessoa. a) I b) II c) III d) I e II e) II e III 10. A alternativa em que aparece uma palavra incorretamente grafada : a) pretensioso, quisesse, catlise b) asceno, mexerico, jil c) exceo, sarjeta, acesso d) assessor, prazeroso, marquesa e) encaixar, pesquisar, surdez

Resoluo: 01. E 02. D 04. B 05. E 07. sesso, seo (ou seco), cesso. 08. D 09. C Exerccios sobre concordncia pronominal Leia o artigo: Pronomes 03. E 06. B 10. B

Questes: 01. (PUC MG) Segundo a norma culta, correto mudar a posio do pronome oblquo tono em: a) Nestes como em outros pontos, assemelham-se aos ingleses. Nestes como em outros pontos, se assemelham aos ingleses. b) Sabe que da lhe esto vindo novas foras de progresso. Sabe que da esto vindo-lhe novas foras de progresso. c) Isto coisa que hoje o envergonharia. Isto coisa que hoje envergonharia-o. d) com prudncia que se servem das inovaes da tcnica. com prudncia que servem-se das inovaes da tcnica. e) Formas anacrnicas de viver e de pensar no se colidem. Formas anacrnicas de viver e de pensar no colidem-se. 02. (STA. CASA) Quando chamar tem sentido de qualificar, pode-se construir o perodo, por exemplo, com objeto direto mais predicativo. Tudo se observa na alternativa: a) Joo alto, mas treinador nenhum chamou-o para jogar. b) Era a viva a chamar pelo falecido. c) Os inimigos chamam-lhe de traidor do povo. d) Chamei pelo colega em voz alta. e) Alguns chamavam-no de fiscal. 03. (UFAC) Assinale a alternativa em que h um grave erro de colocao pronominal. a) No sei porque te hei de seguir os passos. b) Os exames, traga-lhos agora mesmo. c) Ela no se conteve. d) Direi-lhe tudo o que sei. e) O que se no deve fazer. 04. (UFV-MG) Todas as frases esto corretas quanto colocao dos pronomes oblquos tonos, exceto: a) Se o tivesse encontrado, eu lhe teria dito tudo. b) Os alunos tinham preparado-se para a grande prova. c) Em se tratando de caso urgente, nada o retinha em casa. d) No porto de entrada da cidade lia-se, em letras garrafais numa placa de bronze: ESTRANHOS, AFASTEM-SE! e) Logo que me formar, colocar-me-ei disposio da empresa. 05. (PUC-PR) Assinale o perodo em que a colocao do pronome tono pode ser alterada.

a) Passe-me o livro, por favor! b) Foi este o artigo que vocs leram e me recomendaram? c) A criancinha veio, mal se equilibrando nos pezinhos. d) Ter-se-o retirado, quando voc chegar. e) No lhe quero falar sobre o caso.

Resoluo: 01. E 04. D 02. B 05. B 03. A

Exerccios sobre crase Leia o artigo: Crase Questes: 01. Assinale a alternativa em que o uso da crase obrigatrio: a) Um rapazito de palet entrou na rua e foi perguntar Machona pela Nh Rita. (Alusio Azevedo) b) Jos Cndido no tinha nem a cor nem o ttulo convenientes sua filha. (R. Braga) c) Mas o peru se adiantava at beira da mata. (G. Rosa) d) Todos, s vezes, precisam ficar bbados, e por isso bebem. (R. Braga) e) (...) evitei acompanhar Dr. Siqueira em suas visitas vespertinas nossa bem amada. (J. Amado) 02. Qual das alternativas completa corretamente os espaos vazios? I. E entre o sono e o medo, ouviu como se fosse de verdade o apito de um trem igual ____ que ouvira em Limoeiro. (J. Lins do Rego) II. Habituara-se ______ boa vida, tendo de um tudo, regalada. (J. Amado) III. Depois do meu telegrama (lembram: o telegrama em que recusei duzentos mil-ris ___ (pirata), a "Gazeta" entrou a difamar-me. (G. Ramos) IV. Os adultos so gente crescida que vive sempre dizendo pra gente fazer isso e no fazer _____. (Millr Fernandes) a) quele, aquela, aquele, aquilo b) quele, quela, aquele, aquilo c) quele, quela, aquele, quilo d) quele, quela, quele, aquilo e) aquele, quela, aquele, aquilo 03. (CESCEM) Sentou-se ___ mquina e ps-se ___ reescrever uma ___ uma as pginas do relatrio. a) a / a / b) a / / c) / a / a d) / / e) / / a 04. (FASP) Assinale a alternativa com erro de crase:

a) Voc j esteve em Roma? Eu irei Roma logo. b) Refiro-me Roma antiga, na qual viveu Csar. c) Fui Lisboa de meus avs, pois gosto da Lisboa de meus avs. d) J no agrada ir Braslia. A gasolina... e) nenhuma das alternativas est errada. 05. (ESAN) Das frases abaixo, apenas uma est correta, quanto crase. Assinale-a: a) Devemos aliar a teoria prtica. b) Daqui duas semanas ele estar de volta. c) Puseram-se discutir em voz alta. d) Dia dia, a empresa foi crescendo. e) Ele parecia entregue tristes cogitaes. 06. (ABC - MED.) Nas alternativas que seguem, h trs frases, que podem estar corretas ou no. Leia-as atentamente e marque a resposta certa: I. O seu egosmo s era comparvel sua feira. II. No pde entregar-se s suas iluses. III. Quem se vir em apuros, deve recorrer justia. a) Apenas a frase I est correta. b) Apenas a frase II est correta. c) Apenas as frases I e II esto corretas. d) Apenas as frases II e III esto corretas. e) As trs frases esto corretas. 07. (FUND. LUSADA) Assinale a alternativa que completa corretamente o perodo: ____ noite estava clara e os namorados foram _____ praia ver a chegada dos pescadores que voltavam ____ terra. a) / / b) A / / c) A / a / d) / a / e) A / / a 08. (ITA) Analisando as sentenas: I. A vista disso, devemos tomar srias medidas. II. No fale tal coisa as outras. III. Dia a dia a empresa foi crescendo. IV. No ligo aquilo que me disse. Podemos deduzir que: a) Apenas a sentena III no tem crase. b) As sentenas III e IV no tm crase. c) Todas as sentenas tm crase. d) Nenhuma sentena tem crase. e) Apenas a sentena IV no tem crase. 09. (ABC - MED.) A alternativa em que o acento indicativo de crase no procede : a) Tais informaes so iguais s que recebi ontem. b) Perdi uma caneta semelhante sua.

c) A construo da casa obedece s especificaes da Prefeitura. d) O remdio devia ser ingerido gota gota, e no de uma s vez. e) No assistiu a essa operao, mas de seu irmo. 10. (FUVEST) Indique a forma que no ser utilizada para completar a frase seguinte: "Maria pediu ____ psicloga que ____ ajudasse ____ resolver o problema que ___ muito ____ afligia." a) preposio (a) b) pronome pessoal feminino (a) c) contrao da preposio a e do artigo feminino a () d) verbo haver indicando tempo (h) e) artigo feminino (a)

Resoluo: 01. D 02. D 03. C 04. C 05. A 06. E 07. E 08. A 09. D 10. E Exerccios sobre preposio Leia o artigo: Preposio

Questes: 01. (PUC-SP) Use o sinal de crase, se necessrio: a) No vai a festas nem a reunies. b) Chegamos a Universidade as oito horas. 02. (FUVEST) No final da Guerra Civil americana, o ex-coronel ianque (...) sai caa do soldado desertor que realizou assalto a trem com confederados. (O Estado de S. Paulo, 15/09/95) O uso da preposio com permite diferentes interpretaes da frase acima. a) Reescreva-a de duas maneiras diversas, de modo que haja um sentido diferente em cada uma. b) Indique, para cada uma das reaes, a noo expressa da preposio com. 03. (UFPA) No trecho: (O Rio) no se industrializou, deixou explodir a questo social, fermentada por mais de dois milhes de favelados, e inchou, exausto, uma mquina administrativa que no funciona..., a preposio a (que est contrada com o artigo a) traduz uma relao de: a) fim b) causa c) concesso d) limite e) modo 04. (INATEL) Assinale a alternativa em que a norma culta no aceita a contrao da preposio de: a) Aos prantos, despedi-me dela. b) Est na hora da criana dormir.

c) Falava das colegas em pblico. d) Retirei os livros das prateleiras para limp-los. e) O local da chacina estava interditado. 05. Assinale a alternativa em que a preposio destacada estabelea o mesmo tipo de relao que na frase matriz: Criaram-se a po e gua. a) Desejo todo o bem a voc. b) A julgar por esses dados, tudo est perdido. c) Feriram-me a pauladas. d) Andou a colher alguns frutos do mar. e) Ao entardecer, estarei a. 06. (CESGRANRIO) Assinale a opo em que a preposio com traduz uma relao de instrumento: a) "Teria sorte nos outros lugares, com gente estranha." b) "Com o meu av cada vez mais perto de mim, o Santa Rosa seria um inferno." c) "No fumava, e nenhum livro com fora de me prender." d) "Trancava-me no quarto fugindo do aperreio, matando-as com jornais." e) "Andavam por cima do papel estendido com outras j pregadas no breu." 07. (FAU - SANTOS) "O policial recebeu o ladro a bala. Foi necessrio apenas um disparo; o assaltante recebeu a bala na cabea e morreu na hora." No texto, os vocbulos em destaque so respectivamente: a) preposio e artigo b) preposio e preposio c) artigo e artigo d) artigo e preposio e) artigo e pronome indefinido 08. (UNIMEP) "Depois a me recolhe as velas, torna a guard-las na bolsa.", os vocbulos em destaque so, respectivamente: a) pronome pessoal oblquo, preposio, artigo b) artigo, preposio, pronome pessoal oblquo c) artigo, pronome demonstrativo, pronome pessoal oblquo d) artigo, preposio, pronome demonstrativo e) preposio, pronome demonstrativo, pronome pessoal oblquo. 09. (FAC. RUI BARBOSA) Assinale a alternativa em que ocorre combinao de uma preposio com um pronome demonstrativo: a) Estou na mesma situao. b) Neste momento, encerramos nossas transmisses. c) Daqui no saio. d) Ando s pela vida. e) Acordei num lugar estranho. 10. (FECAP) Classifique a palavra como nas construes seguintes, numerando, convenientemente, os parnteses. A seguir, assinale a alternativa correta: 1) Preposio

2) Conjuno Subordinativa Causal 3) Conjuno Subordinativa Conformativa 4) Conjuno Coordenativa Aditiva 5) Advrbio Interrogativo de Modo ( ( ( ( ( ) Perguntamos como chegaste aqui. ) Percorrera as salas como eu mandara. ) Tinha-o como amigo. ) Como estivesse muito frio, fiquei em casa. ) Tanto ele como o irmo so meus amigos.

a) 2 - 4 - 5 - 3 - 1 b) 4 -5 - 3 - 1 - 2 c) 5 - 3 - 1 - 2 - 4 d) 3 - 1 - 2 - 4 - 5 e) 1 - 2 - 4 - 5 - 3 Resoluo: 01 - a) --------b) Chegamos a Universidade s oito horas. 02 - a) 1. No final da Guerra Civil americana, o ex-coronel ianque (...) sai caa do soldado desertor que realizou assalto a trem que levava confederados. 2. No final da Guerra Civil americana, o ex-coronel ianque (...) sai caa do soldado desertor, que, com confederados, realizou assalto a trem. b) Na frase 1, com indica a relao continente-contedo, (trem-soldados), como em copo com gua. Na frase 2, com indica em companhia de. Em 1, com introduz um adjunto adnominal (de trem); em 2, introduz um adjunto adverbial de companhia. 03 - E 07 - A 04 - B 08 - B 05 - C 09 - B 06 - C 10 - C

Qumica
Exerccios sobre a estrutura do tomo Leia o artigo: tomo

Questes: 01. Qual o nmero de tipos de molculas diferentes em um grande reservatrio de gs cloro? Dados: a) O gs cloro tem forma molecular Cl2, isto , ele tem dois tomos de cloro por molcula; b) O cloro apresenta dois istopos naturais: Cl35 e Cl37.

a) 26 tomos de ferro de nmero de massa 56. b) 26 tomos grama de ferro de nmero de massa 56. c) Um isbaro de ferro de nmero de massa 56. d) Um istono de ferro de nmero de massa 56. e) Istopo de ferro de nmero de massa 56.

03. Um tomo de nmero atmico Z e nmero de massa A: a) tem A nutrons. b) tem A eltrons. c) tem Z prtons. d) tem A Z nutrons. e) tem Z eltrons. 04. (STA. CASA) A questo deve ser respondida de acordo com o seguinte cdigo: A teoria de Dalton admitia que: I. tomos so partculas discretas de matria que no podem ser divididas por qualquerprocesso qumico conhecido; II. tomos do mesmo elemento qumico so semelhantes entre si e tm mesma massa; III. tomos de elementos diferentes tm propriedades diferentes. a) Somente I correta. b) Somente II correta. c) Somente III correta. d) I, II, III so corretas. e) I e III so corretas. 05. (FUVEST) O tomo constitudo de 17 prtons, 18 nutrons e 17 eltrons, possui nmero atmico e nmero de massa igual a: a) 17 e 17 b) 17 e 18 c) 18 e 17 d) 17 e 35 e) 35 e 17 06. (FUVEST) A seguinte representao , X = smbolo do elemento qumico, refere-se a tomos com: a) Igual nmero de nutrons; b) Igual nmero de prtons; c) Diferentes nmeros de eltrons; d) Diferentes nmeros de atmicos; e) Diferentes nmeros de oxidao; 07. (MACK) Indique a alternativa que completa corretamente as lacunas do seguinte perodo: Um elemento qumico representado pelo seu ___________ , identificado pelo nmero de __________ e pode apresentar diferente nmero de __________ . a) nome prtons nutrons. b) nome eltrons nutrons. c) smbolo eltrons nutrons. d) smbolo prtons nutrons. e) smbolo eltrons nutrons. 08. (PUC) Quando um metal cristaliza no sistema cbico de faces centradas, seu nmero de coordenao, isto , o nmero de tomos que envolve cada tomo, ser igual a:

a) 3 b) 4 c) 6 d) 8 e) 12 09. (CESCEM) As estruturas cristalinas dos metais A e B so do tipo hexagonal. Essas estruturas devem ter iguais: a) Densidades. b) Nmeros de coordenao. c) Condutibilidades eltricas. d) Propriedades qumicas. e) Nmeros de tomos por volume unitrio. 10. (ENG. SANTOS) As grandes cristalinas das trs substncias sulfato de potssio (K2SO4), enxofre (S) e zinco (Zn) apresentam respectivamente em seus ns: a) ons, molculas e tomos. b) ons, tomos e molculas. c) Molculas, tomos e ons. d) tomos, molculas e ons. e) n.d.a.

Resoluo: 01. Cada Cl2 formado por dois tomos quaisquer de cloro: Cl35 Cl35 Cl35 Cl37 Cl37 Cl37 Essas molculas so quimicamente iguais, porm fisicamente diferentes. Resp: Fisicamente temos 3 tipos de molculas e quimicamente temos um nico tipo de molcula. 02. E 03. D 04. D 05. D 06. B 07. D 08. A 09. B 10. A Exerccios sobre ligaes qumicas e frmulas estruturais Leia o artigo: Isomeria

Questes: 01. Com referncia molcula H2S, fornea: a) Distribuio eletrnica fundamental de cada elemento (H = 1; S = 16) b) Frmula eletrnica. 02. (PUC) Os eltrons que diferenciam o clcio (Z = 20) de seu ction bivalente esto situados no subnvel: a) 3s b) 3p c) 4s d) 3d e) 4p

03. (UFRS) Para a formao da ligao, duas condies so necessrias: um par de eltrons com spins opostos e um orbital estvel em cada tomo. A fora de ligao qualitativamente proporcional interpenetrao das nuvens de carga dos dois tomos. O texto refere-se ligao: a) inica b) metlica c) covalente d) por foras de Van der Waals e) por pontes de hidrognio 04. Um orbital que pertence a um s tomo denomina-se: a) orbital atmico b) orbital s c) orbital molecular d) orbital sigma e) orbital pi A questo 05 refere-se ao texto: Chamaremos de orbital ligante, de maneira simplificada, ao orbital que possui um nico eltron e que entrar em uma ligao covalente. 05. O carbono, no estado fundamental, apresenta um nmero de orbitais ligantes igual a: a) 1 b) 2 c) 3 d) 4 e) 5 A questo 06 refere-se ao texto: Chamaremos de orbital ligante, de maneira simplificada, ao orbital que possui um nico eltron e que entrar em uma ligao covalente. 06. No cloro, em sua configurao fundamental, o orbital ligante do tipo: a) s b) p c) d d) f e) s ou p 07. O hlio, em sua configurao normal, apresenta orbitais ligantes em nmero de: a) 0 b) 1 c) 2 d) 3 e) 4

08. A figura ao lado representa uma ligao:

a) sigma (s s) b) sigma (p p) c) sigma (s p) d) pi e) sigma ou pi 09. A figura abaixo representa uma ligao:

a) sigma (s p) b) pi c) pi ou sigma d) sigma (p p) e) sigma (s s) A questo 10 deve ser respondida pelas alternativas: a) sigma (s s) b) sigma (s p) c) sigma (p p) d) s (p p) e pi 10. A molcula de H2 tem ligao:

Resoluo: 01. a) 1H 1s1 16S 1s2 2s2 2p6 3s2 3p4 == b) H S H 02. C 03. C 04. A 05. B 06. B 07. A 08. C 09. D 10. A Exerccios sobre istonos, isbaros e istopos Leia o artigo: Istopos Isbaros e Istonos

Questes: 01. (ABC) O deutrio um: a) Isbaro de hidrognio. b) Istopo de hidrognio. c) Radioistono do hidrognio. d) Ismero do hidrognio. e) Altropo do hidrognio.

a) istopos. b) altropos. c) isbaros. d) ismeros. e) istopos. 03. (ITA) So definidas quatro espcies de tomos neutros em termos de partculas nucleares: tomo I possui 18 prtons e 21 nutrons tomo II possui 19 prtons e 20 nutrons tomo III possui 20 prtons e 19 nutrons tomo IV possui 20 prtons e 20 nutrons Pode-se concluir que: a) os tomos III e IV so isbaros; b) os tomos II e III so isoeletrnicos; c) os tomos II e IV so istopos; d) os tomos I e II pertencem ao mesmo perodo da Classificao Peridica; e) os tomos II e III possuem o mesmo nmero de massa. 04. (MACK) Assinale a alternativa incorreta:

b) Istopos so tomos de diferentes nmeros atmicos e iguais nmero de nutrons.

d) Istonos so tomos de elementos diferentes e iguais nmero de nutrons. e) n.d.a.

05. (PUC-RIO) Os istopos a) 8, 8, 8 b) 8, 9, 10 c) 16, 17, 18 d) 24, 25, 26 e) 18, 17, 16

possuem respectivamente os seguintes nmeros de nutrons:

06. (PUC RIO) Os fenmenos isotopia, isobaria e alotropia so representados respectivamente pelos exemplos: istopos a) O2; O3 b) c) O2; O3 O2; O3 isbaros altropos

d) e)

O2; O3 O2; O3

07. Tm-se os seguintes tomos e ons genricos:

So, respectivamente, isoeletrnicos, istopos, isbaros, istonos e pertencem ao mesmo elemento qumico os seguintes pares: a) B1+ e E2+ / A e D / C e F / B e E / A e D b) B1+ + E2+ / C e F / A e D / C e B / B e D c) A1+ + F / B e C / C e E / B e D / A e D d) A1+ e E2+ / A e D / C e F / B e E / A e D e) C e F / A e D / B e E / A e F / B e C

Resoluo: 01. B 02. C 05. B 06. E Exerccios sobre substncia e mistura Leia o artigo: Estudo das substncias e misturas 03. E 07. A 04. B

Questes: 01. (PUC) Quando um metal cristaliza no sistema cbico de faces centradas, seu nmero de coordenao, isto , o nmero de tomos que envolvem cada tomo, ser igual a: a) 3 b) 4 c) 6 d) 8 e) 12 02. (CESCEM) As estruturas cristalinas dos metais A e B so do tipo hexagonal. Essas estruturas devem ter iguais: a) Densidades. b) Nmeros de coordenao. c) Condutibilidades eltricas. d) Propriedades qumicas. e) Nmeros de tomos por volume unitrio. 03. (ENG. SANTOS) As grandes cristalinas das trs substncias sulfato de potssio (K2SO4) enxofre (Sa) e zinco (Zn) apresentam respectivamente em seus ns: a) ons, molculas e tomos. b) ons, tomos e molculas. c) Molculas, tomos e ons. d) tomos, molculas e ons. e) n.d.a.

04. (FEI) A alotropia se refere a: a) compostos binrios b) elementos qumicos c) cidos d) bases e) sais 05. (IMES) Assinalar a afirmao correta: a) A ligao dativa igual ligao inica. b) O hidrognio no pode participar de ligao dativa. c) As substncias inicas nas condies ambientes so slidas. d) As substncias moleculares nas condies ambientes so lquidas. e) N.d.a. 06. Tem-se uma mistura de magnsio e bismuto pulverizados. A densidade do magnsio 1,74 g/ml e a do bismuto 9,67 g/ml. Para separar esses dois metais, precisamos escolher um lquido adequado. Assinale a alternativa correta: a) O lquido reage com ambos os metais e tem densidade 2,89 g/ml. b) O lquido reage com um dos metais e tem densidade 2,89 g/ml. c) O lquido no reage com nenhum dos dois metais e tem densidade 2,89 g/ml. d) O lquido reage com um dos metais e tem densidade 1,24 g/ml. e) O lquido no reage com nenhum dos metais e tem densidade 1,24 g/ml. 07. Proponha um procedimento de separao dos componentes de uma mistura de trs substncias A, B e C cujas solubilidades em gua e acetona so indicadas na tabela abaixo: Substncia Solubilidade em gua A B C solvel insolvel insolvel Solubilidade em acetona solvel solvel insolvel

08. As substncias qumicas podem ser representadas por nomes, smbolos ou frmulas. Considerando as seguintes frmulas e smbolos: O2 ; H2O ; H2S ; NaHCO3 ; Fe ; O3 ; H - O - O - H ; H2O2, correto afirmar: (01) H2O H2S e H2O2 representam substncias compotas. (02) O2 e O3 so formas alotrpicas do oxignio. (04) A frmula H2S indica que se trata de uma molcula linear. (08) A gua oxigenada pode ser representada por H - O - O - H ou H2O2, mas esta ltima representao no mostra como os tomos esto ligados. (16) Uma das frmulas representa o hidrogenocarbonato de sdio. (32) Fe pode representar o elemento ferro, um tomo de ferro ou a substncia metlica ferro, embora neste ltimo caso represente um conjunto de nmero indefinido de tomos do elemento ferro unidos por ligao metlica. 09. Ar Naftaleno Lato Gs carbnico Iodo Ouro 18 quilates

Se esses materiais forem classificados em substncias puras e misturas, pertencero ao grupo das substncias puras: a) ar, gs carbnico e lato b) iodo, ouro 18 quilates e naftaleno c) ar, ouro 18 quilates e naftaleno d) ar, ouro 18 quilates e naftaleno e) gs carbnico, naftaleno e iodo 10. (UFPI) Adicionando-se excesso de gua mistura formada por sal de cozinha, areia e acar, obtm-se um sistema: a) homogneo, monofsico; b) homogneo, bifsico; c) heterogneo, monofsico; d) heterogneo, bifsico; e) heterogneo, trifsico.

Resoluo: 01. A 02. B 03. A 04. B 05. C 06. C 07. Um procedimento seria adicionar acetona mistura das trs substncias A, B e C, sendo que A e B iriam se dissolver e C que insolvel, poderia ser separado da mistura por filtrao. A seguir, evaporaramos a acetona e adicionaramos gua ao sistema formado agora pelas substncias A e B; sendo que pelo fato de B ser insolvel, em gua, poderamos separ-la por filtrao, e atravs de uma destilao simples ou evaporao, recuperaramos a substncia A. 08. (01) Correto (02) Correto (04) Errado ( angular) (08) Correto (16) Correto (NaHCO3) (32) Correto 09. E 10. D Exerccios sobre xido - reduo Leia o artigo: Oxidao e reduo

Questes: 01. (UFSC) O nmero de oxidao do chalcognio (O, S, Se, Te, Po) nos compostos H2O2, HMnO4, Na2O4 e F2O so respectivamente: a) 1, -2, -2, -0,5 b) 1, -2, -0,5, +2 c) 2, -2, -2, -2 d) 0,5, +2, -1, +2 e) 1, -0,5, +1, +2 02. (UFES) Considere o composto qumico fosfato de clcio, tambm chamado de ortofosfato de clcio. Em relao a ele, marque a opo incorreta:

(Ca = 40, P = 31, O = 16) a) sua frmula contm 13 (treze tomos); b) a massa de seu mol de 310 g; c) o nmero de oxidao do fsforo +5; d) o composto um sal normal; e) o clcio substitui os dois hidrognios ionizveis do cido de origem. 03. Explique porque a frase d certa ou errada. 04. (ITA) Dadas as substncias abaixo, em qual delas o n de oxidao do mangans mximo? I. MnO2 II. Mn III. MnSO4 IV. K2MnO4 V. KMnO4 a) I b) II c) II d) IV e) V 05. (UEMT) A soma algbrica dos nmeros de oxidao do iodo nas seguintes substncias: hipoiodito de sdio, iodeto de sdio, iodato de amnio e iodo elementar : a) 3 b) 4 c) 5 d) 6 e) 7 06. (UFSE) Calcule o nmero de oxidao do cloro nos compostos: a) HCl b) HClO c) HClO2 d) Ba(ClO3)2 e) Al(ClO4)3 07. (MACK) Assinale o nmero de oxidao INCORRETO: a) Li = -1 b) N = +5 c) S = -2 d) Cl = -1 e) Sr = +2 08. (GV) Os nmeros de oxidao do cromo nos compostos K2Cr2O7, K2CrO4 e Cr2(SO4)3 so respectivamente: a) 6, 4, 3 b) 3, 4, 3

c) 6, 6, 3 d) 3, 3, 3 e) 6, 3, 6 09. (OSEC) Qual das reaes abaixo uma reao de oxi-reduo? a) H3PO4 + NaOH NaH2PO4 + H2O b) CaCO3 CaO + CO2 c) 2 NH3 N2 + 3 H2 d) HNO3 + H2O H3O+ + NO e) AgNO3 + NaCl AgCl + NaNO3 Para responder a pergunta 10, considere as seguintes reaes qumicas: 1. 2 FeSO4 + 2 Ce(SO4)2 + Fe2(SO4)3 + Ce2(SO4)3 2. AgNO3 + NaCl AgCl + NaNO3 3. CuSO4 + 4 NH4OH Cu(NH3)4SO4 + 4 H2O 4. Al(OH)3 + 3 HCl AlCl3 + 3 H2O 5. H2S + 8 HNO3 H2SO4 + 8 NO2 + 4 H2O 10. (FMU) Assinale qual dessas reaes qumicas de neutralizao: a) 1 b) 2 c) 3 d) 4 e) 5

Resoluo: 01. B 02. E 03. Certa. O sal no apresenta H+ nem OH-. Se tivesse H+ seria um sal cido e OH- um sal bsico. 04. E 05. C 06. a) 1 b) +1 c) +3 d) +5 e) +7 07. A 08. C 09. C 10. D

Fisica
Exerccios sobre vetores

Leia o artigo: Clculo Vetorial

Questes abertas: 01. Um projtil lanado com uma velocidade de mdulo 20 m/s e formando com o plano horizontal um ngulo de 60. Calcule os componentes horizontal e vertical da velocidade. 02. (INATEL) Dois corpos A e B se deslocam segundo trajetria perpendiculares, com velocidades constantes, conforme est ilustrado na figura adiante.

As velocidades dos corpos medidas por um observador fixo tm intensidades iguais a: VA = 5,0 (m/s) e VB = 12 (m/s). Quanto mede a velocidade do corpo A em relao ao corpo B? Testes: 03. (UnB) So grandezas escalares todas as quantidades fsicas a seguir, EXCETO: a) massa do tomo de hidrognio; b) intervalo de tempo entre dois eclipses solares; c) peso de um corpo; d) densidade de uma liga de ferro; e) n.d.a. 04. (UEPG - PR) Quando dizemos que a velocidade de uma bola de 20 m/s, horizontal e para a direita, estamos definindo a velocidade como uma grandeza: a) escalar b) algbrica c) linear d) vetorial e) n.d.a. 05. (UFAL) Considere as grandezas fsicas: I. Velocidade II. Temperatura III. Quantidade de movimento IV. Deslocamento V. Fora Destas, a grandeza escalar : a) I b) II c) III d) IV e) V

06. (CESGRANRIO) Das grandezas citadas nas opes a seguir assinale aquela que de natureza vetorial: a) presso b) fora eletromotriz c) corrente eltrica d) campo eltrico e) trabalho 07. (FESP) Num corpo esto aplicadas apenas duas foras de intensidades 12N e 8,0N. Uma possvel intensidade da resultante ser: a) 22N b) 3,0N c) 10N d) zero e) 21N 08. (FUND. CARLOS CHAGAS) O mdulo da resultante de duas foras de mdulos F1 = 6kgf e F2 = 8kgf que formam entre si um ngulo de 90 graus vale: a) 2kgf b) 10kgf c) 14kgf d) 28kgf e) 100kgf 09. (UFAL) Uma partcula est sob ao das foras coplanares conforme o esquema abaixo. A resultante delas uma fora, de intensidade, em N, igual a:

a) 110 b) 70 c) 60 d) 50 e) 30 10. (ACAFE) Os mdulos das foras representadas na figura so F1 = 30N, F2 = 20 N e F3 = 10N. Determine o mdulo da fora resultante:

a) 14,2 N b) 18,6 N c) 25,0 N d) 21,3 N

e) 28,1 N Resoluo: 01 - Vx = 10m/s

02 - 13 m/s 03 - C 04 - D 05 - B 07 - C 08 - B 09 - D Exerccios sobre movimento uniforme Leia o artigo: Cinemtica 06 - D 10 - D

Testes: 01. A luz solar gasta 5,0 . 102 s para chegar Terra. O dimetro do Sol da ordem de 1,4 . 106km. Seja 10n a ordem de grandeza do nmero de corpos idnticos ao Sol que cabem no espao entre o Sol e a Terra, com centros na reta que una o centro do Sol ao centro da Terra. O valor den : a) 1 b) 2 c) 3 d) 4 e) 10 02. Abaixo esto representados, exatamente como foram obtidos, 5 pedaos de fita, marcados por uma "campainha" que os fere periodicamente e com uma freqncia constante. Estas fitas foram puxadas pela mo, no sentido assinalado, representando, portanto, a velocidade da mo do observador.

Qual das fitas representa, no intervalo de tempo considerado (10 tiques), o movimento que tem velocidade escalar mdia maior? a) I b) II c) III d) IV e) V 03. (FUND. CARLOS CHAGAS) Um trem de 200m de comprimento, com velocidade escalar constante de 60 km/h, gasta 36s para atravessar completamente uma ponte. A extenso da ponte, em metros, de: a) 200 b) 400 c) 500 d) 600 e) 800 Para as questes 04 e 05

Dois pontos materiais A e B caminham sobre uma mesma reta e no mesmo sentido. na origem dos tempos a distncia entre os pontos de 5,0 km. A velocidade escalar de A de 80 km/h e a velocidade escalar de B de 60 km/h, mantidas constantes.

04. A velocidade escalar de A relativa a B igual a: a) zero; b) 80 km/h; c) -20 km/h d) 20 km/h e) -80 km/h 05. A encontra B: a) no instante t = 15 h; b) no instante t = 15 min; c) no instante t = 1/4 min; d) nunca e) n.d.a Para as questes 06 e 07 Dois pontos materiais A e B caminham sobre uma mesma reta e no mesmo sentido. na origem dos tempos a distncia entre os pontos de 5,0 km. A velocidade escalar de A de 80 km/h e a velocidade escalar de B de 60 km/h, mantidas constantes.

06. A funo horria que descreve o movimento de B, relativo a A para s em km e t em h, representada por: a) s = 5,0 - 20t b) s = 5,0 + 20 t c) s = 20t d) s = -20t e) n.d.a. 07. A funo horria que descreve o movimento de A, relativo a B para s em km e t em h, representada por: a) s = -5,0 - 20t b) s = 5,0 + 20 t c) s = 20t - 5,0 d) s = -20t e) n.d.a. 08. Considere dois trens T1 e T2 caminhando em linhas frreas retilneas e paralelas com velocidades de intensidade V1 = 36 km/h e V2 = 72 km/h, em sentidos opostos. Um observador O1 est no trem T1 e nota que a passagem de T2, diante de sua janela, durou um intervalo de tempo de 10s. O tempo gasto por T2, para passar por um tnel de comprimento 200m, de: a) 25s b) 20s c) 30s

d) 50s e) 60s Para as questes 09 e 10 Considere um movimento cuja posio s, em funo do tempo t, est representado no grfico.

09. A distncia percorrida pelo mvel entre os instantes t = 0 e t = 20s, em metros, vale: a) -40 b) zero c) 20 d) 40 e) 80 10. O mvel passa pela origem no instante: a) zero b) 5,0s c) 10s d) 15s e) 20s Resoluo: 01 - B 02 - A 03 - B 04 - D 06 - A 07 - C 08 - A 09 - E Exerccios sobre movimento uniforme variado Leia o artigo: Movimento Uniformemente Variado 05 - B 10 - C

Testes: 01. (FUVEST) Um veculo parte do repouso em movimento retilneo e acelera com acelerao escalar constante e igual a 2,0 m/s2. Pode-se dizer que sua velocidade escalar e a distncia percorrida aps 3,0 segundos, valem, respectivamente: a) 6,0 m/s e 9,0m; b) 6,0m/s e 18m; c) 3,0 m/s e 12m; d) 12 m/s e 35m; e) 2,0 m/s e 12 m 02. (FUND. CARLOS CHAGAS) Dois mveis A e B movimentam-se ao longo do eixo x, obedecendo s equaes mvel A: xA = 100 + 5,0t e mvel B: xB = 5,0t2, onde xA e xB so medidos em m e t em s. Podese afirmar que: a) A e B possuem a mesma velocidade; b) A e B possuem a mesma acelerao; c) o movimento de B uniforme e o de A acelerado; d) entre t = 0 e t = 2,0s ambos percorrem a mesma distncia;

e) a acelerao de A nula e a de B tem intensidade igual a 10 m/s2. 03. (MACKENZIE) Um mvel parte do repouso com acelerao constante de intensidade igual a 2,0 m/s2 em uma trajetria retilnea. Aps 20s, comea a frear uniformemente at parar a 500m do ponto de partida. Em valor absoluto, a acelerao de freada foi: a) 8,0 m/s2 b) 6,0 m/s2 c) 4,0 m/s2 d) 2,0 m/s2 e) 1,6 m/s2 04. (UFMA) Uma motocicleta pode manter uma acelerao constante de intensidade 10 m/s2. A velocidade inicial de um motociclista, com esta motocicleta, que deseja percorrer uma distncia de 500m, em linha reta, chegando ao final desta com uma velocidade de intensidade 100 m/s : a) zero b) 5,0 m/s c) 10 m/s d) 15 m/s e) 20 m/s 05. (UFPA) Um ponto material parte do repouso em movimento uniformemente variado e, aps percorrer 12 m, est animado de uma velocidade escalar de 6,0 m/s. A acelerao escalar do ponto material, em m/s vale: a) 1,5 b) 1,0 c) 2,5 d) 2,0 e) n.d.a. 06. (UNIP) Na figura representamos a coordenada de posio x, em funo do tempo, para um mvel que se desloca ao longo do eixo Ox.

Os trechos AB e CD so arcos de parbola com eixos de simetria paralelos ao eixo das posies. No intervalo de tempo em que o mvel se aproxima de origem dos espaos o seu movimento : a) uniforme e progressivo; b) retrgrado e acelerado; c) retrgrado e retardado; d) progressivo, retardado e uniformemente variado; e) progressivo, acelerado e uniformemente. 07. (PUCC) Um vaso de flores cai livremente do alto de um edifcio. Aps ter percorrido 320cm ele passa por um andar que mede 2,85 m de altura. Quanto tempo ele gasta para passar por esse andar? Desprezar a resistncia do ar e assumir g = 10 m/s2. a) 1,0s b) 0,80s c) 0,30s d) 1,2s

e) 1,5s 08. (PUCC) Duas bolas A e B, sendo a massa de A igual ao dobro da massa de B, so lanadas verticalmente para cima, a partir de um mesmo plano horizontal com velocidades iniciais. Desprezando-se a resistncia que o ar pode oferecer, podemos afirmar que: a) o tempo gasto na subida pela bola A maior que o gasto pela bola B tambm na subida; b) a bola A atinge altura menor que a B; c) a bola B volta ao ponto de partida num tempo menor que a bola A; d) as duas bolas atingem a mesma altura; e) os tempos que as bolas gastam durante as subidas so maiores que os gastos nas descidas. 09. (UFPR) Um corpo lanado verticalmente para cima, atinge certa altura, e desce. Levando-se em conta a resistncia do ar, pode-se afirmar que o mdulo de sua acelerao : a) maior, quando o corpo estiver subindo; b) maior, quando o corpo estiver descendo; c) igual ao da acelerao da gravidade, apenas quando o corpo estiver subindo; d) o mesmo, tanto na subida quanto na descida; e) igual ao da acelerao da gravidade, tanto na subida quanto na descida. 10. (UCPR) Num local onde a acelerao da gravidade vale 10 m/s2 uma pedra abandonada de um helicptero no instante em que este est a uma altura de 1000m em relao ao solo. Sendo 20s o tempo que a pedra gasta para chegar ao solo, pode-se concluir que no instante do abandono da pedra o helicptero: (Desprezam-se as resistncias passivas) a) subia b) descia c) estava parado d) encontrava-se em situao indeterminada face aos dados; e) esta situao impossvel fisicamente. Resoluo: 01 - A 02 - E 03 - A 04 - A 09 - A 05 - A 10 - A

06 - D 07 - C 08 -D Exerccios sobre reflexo e refrao

Leia o artigo: Reflexo, Difuso e Refrao da luz

Questo: 01. Mediu-se o mdulo da velocidade da luz amarela de sdio propagando-se num slido e obteve-se o valor 2,00 . 108m/s. Qual o ndice de refrao absoluto desse slido, para a luz de sdio? Usar velocidade da luz no vcuo igual a 3,00 . 108m/s.

Testes: 02. (PUC) Quando um feixe de luz monocromtico sofre uma mudana de meio, passando do ar para a gua, a grandeza que se mantm sempre constante : a) o comprimento de onda b) a velocidade de propagao c) a intensidade do feixe d) a direo de propagao e) a freqncia

03. (PUC) Uma exploso solar observada na Terra 500s depois de produzida. Se o espao entre a Terra e o Sol fosse constitudo de um meio de ndice de refrao igual a 2, o tempo decorrido entre o instante da exploso e o de sua observao na Terra seria: a) nulo b) 1 000s c) 250s d) 750s e) o mesmo, pois o que se observa na Terra o barulho produzido pela exploso, cuja velocidade de propagao no tem nenhuma relao com o ndice de refrao do meio. 04. (IME) A diferena entre os comprimentos de onda de um raio luminoso no ar e em um meio de ndice de refrao 1,6 de 3 000C. Qual o comprimento de onda no ar? a) 2000C b) 4000C c) 6000C d) 8000C e) 10000C 05. Considere um feixe de luz monocromtica proveniente do vcuo incidindo normalmente sobre a superfcie plana de um bloco de vidro de ndice de refrao absoluto 1,5. Uma parcela da luz incidente refletida, retornando para o vcuo, enquanto que outra parcela refratada, passando a propagar-se no vidro. No diagrama abaixo, o ponto P caracteriza a luz incidente, cujo sentido de propagao foi adotado como positivo.

Dos pontos numerados de I a IV, os que caracterizam, respectivamente, a luz refletida e a luz refratada so: a) I e III b) II e III c) I e IV d) II e IV e) III e IV 06. Um feixe cilndrico de luz monocromtica, propagando-se no ar, incide na superfcie da gua de um tanque, originando dois novos feixes: um refletido e outro refratado. A respeito dessa situao, podemos afirmar que: a) o mdulo da velocidade de propagao da luz refletida menor que o da luz refratada. b) A freqncia da luz refletida maior que a da luz refratada. c) O ngulo de reflexo menor que o de refrao. d) O comprimento de onda da luz refletida maior que o da luz refratada. e) O comprimento de onda da luz refletida igual ao da luz refratada. 07. (UFMG) A figura mostra um feixe de luz que passa do vidro para a gua.

Com relao a essa situao, correto afirmar que: a) A freqncia da luz maior no vidro do que na gua. b) O mdulo da velocidade da luz no vidro maior do que na gua. c) O comprimento de onda da luz no vidro menor do que na gua. d) O ndice de refrao absoluto do vidro menor do que o ndice de refrao absoluto da gua. e) O perodo da luz maior na gua do que no vidro. 08. Em determinadas condies, pode-se ouvir o eco de um som. O fenmeno acstico que explica o eco : a) a refrao b) a reflexo c) a ressonncia d) a interferncia e) a difrao 09. (UFC) O ouvido humano percebe distintamente dois sons quando estes esto separados por um intervalo de tempo mnimo de 0,10s. Uma pessoa emite um som breve e forte que se reflete num anteparo situado a uma distncia d. O mnimo valor de d para que a pessoa perceba com distino o eco : a) 85m b) 68m c) 51m d) 34m e) 17m 10. Um dos discos clssicos do rock, o lbum The Dark Side of the Moon, do grupo inglsPink Floyd, lanado em 1973, traz em sua capa uma bonita figura da luz branca sendo decomposta em um prisma ptico, o que caracteriza o fenmeno da disperso. Pelo que se conclui da ilustrao, o prisma de vidro (ou de material semelhante) e est imerso no ar.

Cada freqncia do espectro da luz branca sofre um desvio diferente na travessia do prisma, permitindo a obteno de um feixe policromtico no qual se distinguem as cores fundamentais presentes, tambm, num arco-ris. A respeito do fenmeno da disperso da luz no prisma, analise as alternativas abaixo e aponte a correta: a) A cor que mais se desvia a violeta, pois ao refratar-se do ar para o vidro, sofre menor variao de velocidade de propagao que as demais cores. b) A cor que menos se desvia a violeta, pois ao refratar-se do ar para o vidro e do vidro para o ar, sofre maior variao no comprimento de onda que as demais cores.

c) O diferente desvio sofrido por cada uma das cores componentes do espectro da luz branca determinado pelo ndice de refrao que o vidro apresenta para cada freqncia, isto , para a luz violeta ele apresenta maior ndice de refrao que para a luz vermelha. d) Na travessia do prisma, a cor de maior freqncia sofre o menor desvio, enquanto que a cor de menor freqncia sofre o maior desvio. e) O desvio sofrido por cada uma das cores componentes do espectro da luz branca determinado pela variao de freqncia que cada uma delas sofre na refrao do ar para os vidro e do vidro para o ar. Resoluo: 01 - O ndice de refrao absoluto do meio vale 1,50. 02 - E 06 - D 03 - B 07 - C 04 - D 08 - B 05 - C 09 - E

10 - C

Você também pode gostar